Вы находитесь на странице: 1из 130

Hello doctors. I have gathered few recent exam questions since jan 2017 uptill now.

Its just a small


effort to help eachother to get through the prometric exam requirement.
Plz forgive any errors as nothing is done intentionallyits solely to help
Dont forget to do the main files such as SLE SIX PLUS and facebook correction files along with this file
Keep me in your paryers
Dr.shaheen kashif
Exam1
1. Gap between two soldered parts should be:
a. <0.2
b.0.2*****
c.0.4
d.0.6
2.
2. Shade guide for cement during porcelain veneer restoration you must
1.use cement base that is lighter than porcelain*******
2.trying the base without catalyst
3.trying catalyst alone
4.trying the base cement mixed with small amount of catalyst
3.
3. In case of alveolar cleft with cleft palate the in area of cleft:
1/ congenitally missing********
2/normal morphology
3/erupted in contralateral side
4.
. In case of class 5 provisional restoration whats factor important for locking:
1/ 50% expansion of restoration
2/ 25 % polymerization shrinkage
3/ undercut*********
5.
- Root canal treated tooth and exposed to oral environment When you should do reendo after
1-one month*********
2- two month
3- 2 week
4 6 days
6.
What is the type of wax used to verify the occlusal reduction for full veneer restoration
A. onlay wax
B. lowa wax
C. Utility wax*********
D. korecta wax?
7.
The cone beam radiograph is mostly
A. have great radiation than conventional ways
B. can be used as traditional method for pt. assessment*
C. rare and not applicaple
D.it is best way for diagnosis of tmj**********
8.
Exam2
1. The action of the Hawley appliance is mainly
a) intrusion
b) tipping***************
c) bodily movement

2. A light force applied to the periodontal ligament during orthodontic treatment is considered
a. intermittent
b. direct
c. continous******************
d. indirect
3. The fluoride concentration in most dentifrices range from
a. 1-5ppm
b. 900-1500ppm***********************
c. 450-700ppm
d. 4000-6000ppm
4. Why is the surgical stent required for an immediate denture?
a. to give an idea of the anatomy of the region*********************
b. prevent hematoma
c. to determine occlusion
5. Which tooth will the matrix band be a problem with when placing a two surface amalgam?
a. mesial on maxillary first molar**********************
b. distal on maxillary first premolar
c. mesial on maxillary second premilar
d. distal on mandibular first molar
6. When do you do serial extraction?
a. for space deficiency in mandibular anterior region**********************
b. for space deficieny in mandibular posterior region
c. for space deficiency in maxillary anterior region
d. for space deficiency in maxillary posterior region
7. On what surface of the tooth is there deposition of F
a. smooth surface*******************
b. pits
c. fissures
8. Organism implicated on causing severe spreading abscesses include
a. Fusobacterium
b. Campylobacter
c. Enterococci
d. Bacteroides**********************
9. A periodontal exam of a patient referred for endodontic treatment
a. there is an inward flow of fluid
b. there is an outward flow of fluid**************************
c. there is no fluid
10. You fit a new completed denture and the patient complains of cheek biting, what would you do
a. grind buccal of lower teeth***********************
b. gring buccal of upper teeth
c. grind lingula of lower teeth
d. grind lingula of upper teeth
11. When doing an endo treatment you hit a ledge, what are you going to do?
a. use smaller instrument and get beyond the ledge*************************
b. fill as far as you have reamed
c. use a small round bur and remove the ledge
d. continue working gently to eliminate the ledge
12. A patient with new denture can not make the "S" and "TH" sound, what is the problem?
a. extensive vertical overlap
b. incisors placed too far**************************
c. incisors placed too far lingually
13. What composite should ideally be used for a class 5
a. microfil because it is pollished better******************
b. microfil because it is stronger
c. hybrid because it is polished better
d. hybrid because it is stronger
14. What would you warn patient about who is taking birth control pills and required Penicillian
a. penicillin decrease effectivenes of birth control pill**********************
b. birth control pill decrease effectiveness of penicillian
c. they may develop allergy
15. What radiograph would you prescribe for a young patient who has no caries and where the molar teeth
are contacting each other?
a. BW and occlusal ***********************
b. FMS
c. pan
d. pan and ceph
16. How far should implants be placed from one another
a. 3mm******************
b. 4mm
c. 5mm
d. 7mm
17. In what part of the mouth are metastases seen most frequently
a. mandible *********************
b. lateral border of tongue
c. palate
d. floor of the mouth
18. What is the complication of up righting molar
a. move distally and extrudes****************************
b. a class 3 molar relationship can develop
c. class 2 molar relationship can develop
19. When will the BULL rule be utilized with the selective grinding
a. working side***********************
b. balance side
c. protrusive movement
d. all of the above
20. What can make porcelain crown lighter
a. value***********************
b. chroma
c. hue
21. Which of the following are effects common to pentobarbital, diazepam and meperidine
a. amnesia and skeletal muscle relaxation
b. anticonvulsant and hypnotic************************
c. analgesia and relief of anxiety
22. Pan showing lucency going inferior over the body of mandible close to the angle. Informed the patient
was involved in an accident. Identify the lucency
a. pharyngeal airspace
b. fracture**********************
c. artifact-retake radiograph
23. After opening the flap buccally in the maxillary premolar area, how will you suture it
a. interrupted*******************8
b. intermittent
c. matttress
24.Fracture of a rest of a chromium cobalt denture is due to
a. over-finish and polish
b. inadequate occlusal preparation**********************
25. In office bleaching changes the shade through all except
a. dehydration
b. etching tooth***********************
c. oxidation of colorant
d. surface demineralization
26. The lingual root of maxillary first molar radiographically appears mesial to the mesiobuccal root because
the cone was directed from
a. mesial********************
b. distal
c. superior
d. inferior
27. You fit new completed denture and the patient complains of cheek bite, what will you do?
a. grinding buccal of lower teeth*************************
b. grinding buccal of upper teeth
c. grinding lingual of lower teeth
d. grindinging lingual of upper teeth
28. Probing depth differs depending on
a. inflammation of the tissue
b. force used for probing
c. if probing depth decrease, it may be due to reduction of inflammation and swelling, not due to real regain
of attachment*****************************
29. If during the root canal treatment, the dentist breaks the apical seating and extrude the file over the
foramen
a. fill the canal with a master file of a larger number then master cone
b. increase the size of the file and re-create a new apical seal and then fill with the master cone of that
size**************************
c. try to fill in as many Gutta percha points as possible
d. use canal medication instead of filling
30. If removal of torus must be performed to a patient with full-mouth dentition, where shouldthe incision be
made?
a. right on the top of the torus
b. at the base of the torus
c. midline of the torus*********************
d. from the gingival sulcus of the adjacent teeth
31. After implant placement, an edentulous patient should
a. avoid wearing anything for 2 weeks*************************
b. immediately have healing abutments placed over the implants
c. should wear an immediate denture to protect the implant sites
32. After perio surgery, the re-attachment can hapen
a. as soon as in a week*********************
b. to the dentin or cementum
33. There are more detached plaques within supragingival plaques that subgingival plaques. The detached
plaques within subgingival area are the ones that are more toxic to tissue than attached plaques
a. both statements are correct*************************
b. the first statement is correct but not the second
c. the first statement is wrong, but the second statement os correct
d. both statement are wrong
34. TMD patients usually have
a. psychosis*********************
b. antisocial tendecy
c. Schizotypical character
d. drug abuse
35. Why is z-plasty preferred over diamond technique for frenum detachment
a. less scar contraction*******************
b. easier in terms of technique
c. less incision needed, no suture needed
36. What is the purpose of leveling the curve of Spee
a. correct open bite
b. correct deep bite*********************
c. correct angulation of the teeth
d. change arch diameter
37. Palatal expansion device does not need a labial bow because
a. labial bow is not rigid enough
b. labial bow would limit the expansion effect
c. labial bow is not functional in this case
38. What happens with intercanine distance after mixed dentition
a. increased************************
b. decreased
c. stable, no change
39. Upper limb IV injection, the most likely problem is
a. Phlebitis*********************8
b. intra-arterial injection
c. vascularbreakage
40. Which of the following materials is very hard to remove from patient's mouth
a. alginate
b. polyether*************************
c. polysulfide
d. silicone
41. The best and the most effective way to remove stained mottled enamel
a. home bleaching
b. microabrasion technique*****************
c. office bleaching
d. walking bleaching
42. The following have additive action for each component when used together, except for one, what is the
exception
a. Atropine+glycopyrolate (anticholinergic)
b. aspirin acetaminophen
c. penicillin tetracycline******************8888
43. patients who have nausea and vomit during N2O sedation usually
a. are allergic to N20
b. inhale too high concentration of N20*******************
c. eat a big meal right before the proceudre
44. Ataxic epilepsy patients - what are their most common dental problem
a. adontia
b. malocclusion
c. trauma
d. gingival hyperplasia due to medication***********************
45. Porcelain veneer - what is the most important advantage compared with porcelain venee, what is the most
important advantage of resin veneer
a. esthetic
b. cost************************
c. tooth preservation
46. Compared with class II plaster, which one of the following is NOT the characteristics of die strength
a. better compression strength
b. better tensile strength
c. require less water
d. higher expansion**************************
47. Which of the followings is not an advantage of resin based GIC over water based GIC?
a. better fluoride released*********************
b. better bonding
c. better esthetic
d. easier for manipulation
48. What is the characteristic of an implant that would change bony resorption pattern
a. intaosseous integration******************************
b. integrate with bone
49. What are the reccommended numbers of implants for complete edentous patients
a. maxilla 1 mandibular 1
b. maxilla 3 mandibular 2
c. maxilla 6 mandibular 4***********************
d. maxilla 8 mandibular 6
50. Apically positioned flap is almost impossible to perform on
a. maxillary buccal area
b. maxillary lingual area**************************
c. mandibular lingual area
d. mandibular buccal area
51. In order to increase the success rate of treating local juvenile periodontitis, what should be done
a. combine usage of systemic antibiotic
b. local antibiotic treatment*******************
c. use of chlorhexidine
Exam3

1)Which part of instrument should be parallel during calculus removal:


A. Shank***
B. Blade
C. Cutting edge
D. Handle
2) The most non-irritant material (biocompatible) is:
A. Titanium ***
B. Gold noble alloy
C. Chrome (cobalt)
D. Chrome (nickel)
3) Pt. had trauma in facial (blow ) nasal discharge contain CSF what is content:
a. Glucose ***
b. Protein
c. Glucose oxide
d. Tri sulphate
A. Hard caries on dentin is removed by
B. Bur high speed***
C. Bur low speed
D. (Chesile)
4) Verrucous carcinoma:
A. Malignant lesion***
B. Bengin lesion
5) The 4th canal in maxillary first molar found in:
A. In palatal root
B. In MB root ***
C. In DB root
D. Between palatal and distao-buccal canal
6) During intra-uterine life, embryo become fetus after:
A. 1st week
B. 1st month
C. 2nd months
7) 3rd months***
8) Syphilis 1st appear as:
A. Multiple vesicles
B. Erythematous (reaching)
C. Ulcer***
D. (Bullia)
9) The posterior palatal seal of maxillary complete area can be detected by the following except:
A. Hamular notch
B. Fovea palatine***
C. Anterior vibrating line
D. Posterior vibrating line
10) Over extend GP should be treated by:
A. Surgically***
B. Solvent
C. Ultrasonic scalier
D. Round bur rotary inst.
11) Blue grass appliance used for:
A. Tongue thrusting
B. Mouth breathing
C. Thumb sucking***
D. Correct swallowing
12) Question regarding to obturation:
A. Obturation technique affect sealer distribution in coronal
B. Obturation technique affect sealer distribution in middle
C. Obturation technique affect sealer distribution in apical
D. No relation bet. Sealer distribution and location of roott *****************
13) Submandibular abscess drainage through:
A. Incision intaorally through mylohyoid muscle
B. Extra orally at the mandibular angle area****************
14) Gingivectomy indication for all except:
A. Extensive alveolar removal ***************
B. Eliminate supra bony pocket
C. Eliminate periodontal abscess
15) pt .with infection in sublingual , submandibular , sub mental area called:
A. Ludwig's angina ***
16) After scaling and RP , healing done by:
A. Long junctional epithelial*****************
17) Management of fractured tuberosity:
A. Replace & suture the soft tissue ******************
B. Replace & suture with intra-alveolar wire
C. Remove & leave it to heal
D. Remove & suture to heal with primary heal
18) Best feature of sealant should be:
A. Viscosity ***************************8
B. High retentive
C. Resilient
D. High strength
19) New recommendation is to finish amalgam:
A. 24 h**************
20) After gingivectomy , surface epithelial occurs:
A. 3-5 days
B. 7-14 days******************
C. 15-50 days
D. 20-22 days
21) Ugly duckling stage:
A. 6-8 y
B. 9-11y************
22) Open margin on crown what to do:
a. Cover it with cement
b. Replace crown *************
c. Cover it with porcelain
d. Cover it by burnishing the metal
23) 25 file color:
a. Red**************
24) Best pedo behavior:
a. Tell-show-do**************
25) office bleaching:
a. 38% hydrogen peroxide****************
26) Tooth loss by acids :
Erosion*****************
28) Unacceptable attitude from dentist for uncooperative pt. with physical disability:
a. Send him home *********************
b. Tell- show do
c. Hand mouth tech.
29) What is importance of epinephrine in anesthesia:
a. Prolong vasoconstriction**************
30) Small access in the central lead to:
a. Incomplete debridement****************
31) Pt. physiological stress , smoker before 1 y he reported to school he has necrosis gingival , redness, no
attachment loss , treated by mechanical debridement and systemic antimicrobial thereby:
a. Periodontitis
b. ANUG***
c. Lichen planus
d. ANUP
32) Traditional glass ionomer:
a. Susceptible to dehydrate + no free monomer*****************
b. 32)Pedo patient got trauma in one of the anterior teeth causing pulp hyperemia and this pulp hyperemia is
due to:
a. loss of blood
c. b. intrusion of tooth
d. c. congestion of pulp ***************
33)The periodontal tissues comprise which of the following tissues:
a. Gingiva and the PDL
b. Gingival, PDL, and alveolar bone
c. Gingival, PDL, alveolar bone, and cementum***************
d. Gingival, PDL, alveolar bone, cementum, and enamel
34)Treatment of Keratocyts:
a. Through Enucleation / Incision
The relationship between The working end of the hand piece and tooth surface is called:
a. Adaptation **********************
b. Activation
c. Angulations
d. Accessibility
36)Major connector is applied in:
a. Diagnosis and ttt plan******************
37)Delay eruption of the teeth with insomnia and weight loss:
A. Clediocranial dysphagia
B. Hypothyroidism
C. Hypopituitarism *************
38)2 years child came to your clinic complaining from white spot in all his teeth . ttt should include:
A. Parental counseling *******************
B. Caries examination
C. Nutritional survey
D,ANTICIPATORY guidance******************
39))Pt. with pain in lower mandibular molar , he has osteoporosis , the dentist should expect the following in
OPG: ^
A. Cotton wool appearance
B. Orange peel appearance
C. Root may not be apparent due to bone density
D. Thin cortical plates***********************
40)Best instrument to plane gingival area in CLass 2:
A. Gingival marginal trimmer ****************
B. Hatchet
C. Chisel
41)Retrograde filling is indicated in what condition:
A. Max. central incisor with good filling with 9 mm radiolucency ****
B. Max. premolar with post & core & buccal root with 4 mm short filling & RL at the apex but the palatal
root with good filling
C. Max. 1st molar with MBR & DBR short filling & palatal root with fracture instrument
43) During access cavity , it is important to concentrate on:
A. Remove caries before access opening **************
B. Take diagnosis radiographically
C. Use rubber dam
44)Enamel caries proximally:
A. Should be restored immediately
B. Should be restored when reach DEJ *****************
C. Should be restored when reach DEJ and wide spread start
45)H- file has more cutting efficiency compared to k-files due to:
A. More negative rake angle
B. More positive rake angle ******************
C. More cutting tip
46)During eruption of lower mandibular molar , there is a spicule of hard tissue in the central fossa when the
tooth is erupting , this is called:
A. Ectopic eruption
B. Hematoma
C. Eruption sequestering*****************
D. Eruption cyst
47)in systamic water F we look for :
a)age *****************
b)temperature
c) conc.
d)type
48)symptoms of chronic pericoronitis :
a)pain in mouth open
b)halitosis
c) lymph enlargment
d)all**************

49: The body secrets antibody against antigen using which cells:
A. T lymphocyte
B. B lymphocyte*******************
C. Palasma cell
50: What kind of radiograph which we do not use for TMJ movement:
A. Transcranial*******************
B. Computerized tomography
C. Conventional tomography
D. Arthrography
51: The cause of black cast which prevents pickling due to:
A. over heat**********************
B. Contamination with gas
C. incomplete casting
52: Patient has bulimia and has lesion in palatal surface in upper teeth with recurrent vomiting. What is the
type of lesion :
A. Attrition
B. Abrasion
C. Erosion************************
53: Electric pulp tester on the young is not accurate because:
A. Late appearance of Fibers A************************
B. Late appearance of Fibers C
C. Early appearance of fibers A
D. Early appearance of fibers C
54: The most frequent cause of failure of a cast crown restoration is:
A.Failure to extend the crown preparation adequately into the gingival sulcus
B. Lack of attention in carving occlusal anatomy of the tooth***********************
C.Lack of attention to tooth shape, position, and contacts
D.Lack of prominent cusps, deep sulcus, and marginal ridges
55: 8 years old patient has trauma in tooth number 11 , half an hour ago, with incipient pulp exposure .
treatment is:
A. Pulpotomy with formacresol
B. Apexification
C. Direct pulp capping**********************
56: Patient presented to you with immediate denture done 5 - 10 months ago complaining of pain in the
labial area of the mandible what is the diagnosis
A. Epulis fissuratum ******************************
B. Hypertropic labial frenum
57: Patient on treatment with steroids are placed on antibiotic after oral surgical procedure because:
A.The Patient is more susceptible to infection************************
B.Antibiotics are synergistic to steroids
C.Antibiotics protect the Pt. from steroid depletion
58: After class V GIC restoration removal of a thin flush of GIC is done by:
Scaller or knife immediately
Finishing abrasives immediately
Scale or knife subsequently
Finishing abrasives subsequently
1. a + b
2. a + d*************************
3. a + c
4. d + c
59: We can use to score palatal posterior seal on a cast:
1. Le jao carver.
2. Kingsley scraper. *********************
3.lecron carver
60:. Why the moisture heat sterilization is better than dry heat sterilization
a. Breaks the protein cell membrane at moderately low temp. ********************
b. Breaks the protein cell membrane at very high temp.
61: The following factors affect the health:
1- hereditary.
2- environment.
3- social and economic factors.
4- family welfare.
A) 1+2
B) 1+2+4
C) 1+2+3
D) All of the above*********************
62:. Clinical failure of the amalgam restoration usually occurs from:
a. Improper cavity preparation.
b. Faulty manipulation.
c. Both of the above*****************8
63:. Cavity etching before applying GIC is:
1.10% Polyacrylic acid 10 seconds.*****************************
2. Polyacrylic acid 60 seconds.
3. Phosphoric acid 10 seconds.
4. Phosphoric acid 60 seconds
64.After scaling and root planning healing occurs by:
a. Long junctional epithelium. ****************************
b. New attachment.
c. New bone and connective tissue formation.
d. New attached periodontal ligament fibers
65:. The divergence should be mesiodistally for an amalgam restoration:
a. no it should be convergent.
b. if the remaining proximal marginal ridge = 1.6 mm.
c. if the remaining proximal marginal ridge only > 1.6mm.
d. if the remaining proximal marginal ridge only <1.6mm**********************
66- Patient needs fixed bridge after you check in mouth of the patient see change color of bridge to cloudy to
milky what causes?
a. Excessive fired. **********************
b. Reduced fired.
c. Excessive moisture.
d. Increased poursity.

Exam4

1-Endo failure mainly due to which organism :


A. Staphylococcus onerous
B. Actinomycen and entrococcus***
C. Streptococcus
2-8 years old child , trauma of maxillary incisor the material used for first visit Apexcification
A. Ca(OH) + CMCP
B. MTA***
C. ZOE
3-CMCP contain
A. 65 % camphor , 35 % phenoll***
4-CSF , what is contain
A. High protein
B. High glucosee***
5-Over extend GP should be treated by
A. Surgically***
B. Solvent
C. Ultrasonic scalier
D. Round bur rotary instrument
Syphilis oral lesion
A. Multiple vesicles
B. Erythematous reaching
C. Ulcer***
6-5 years old, extracted upper 1st primary molar. what is the space maintainer
A. Band and loop ***
B. Crown and loop
7-Most common marginal failure of fixed prosthodontics is
A. Periodontal
B. Pulpal
C. Caries***
D. Mechanical
8-Pt. came with severe pain from tooth with direct pulp capping , x-ray no PA , no pain with percussion but
the pain with
cold and hot lasting diagnosis
A. Reversible pulpitis
B. Irreversible pulpitis ***
C. Acute apical periodontitis
D. Cracked
9-Sinus track with gutta percha traced to the apex, indicate
A. Horizontal fracture
B. Chronic apical abscess****************
C. Pulp degeneration
D. Acute form of chronic abscess
10-Internal resorption
A. Rarely in deciduous teeth
B. Initiated by odontoblast
C. Seldom confused with external resorption
D. It is usually asymptomatic*****************8
11-When doing re-endo to remove GP , best method is
A. Chemical
B. Mechanical
C. Heat**********
D. Chemically , mechanical
12-Pt. have an accident in aside he has infra orbital level of eyes , psudoptosis and pain to temporofrontal
suture and posterior open bit , which did he has
A. Gorlin fracture
B. Le-fort 1 & 2
C. Le-fort 3
D. Isolated zygomatic fracture ***
13-3 walls defect tilled which autogenous osseous coagulum that contain:
1. Intraoral cancellous bone mixed with patient blood.
2. Intraoral cortical bone mixed with patient blood
3. Mixed intraoral cancellous & cortical bone mixed with patient blood*****************
4. Extra oral cancellous bone mixed with patient blood
14-2 walls defect in perio what is the best graft to treat this defect
a. Cortical freeze dried bone allograft
b. Cancellous freeze dried bone allograft.***************
c. All are the same
15-Bluegrass grass appliance used for
A. Tongue thrusting
B. Mouth breathing
C. Thumb sucking***
D. Correct swallowing
16-Best time treating 4 years old child pt. in dental clinic
A. Morning***
B. Afternoon
C. Evening
D. Late evening
17-Biological width is
A. Epithelial attachment + connective tissue attachment***
18-Glenoid fossa in
A. Temporomandibular jointt ***
19-Composite matrix consist of
A. Bis- GMA***
B. HEMA
C. Celluloid
D. Mylar
20-After Gingivectomy , surface epithelial occurs
A. 3-5 days
B. 7-14 days***
C. 15-50 days
D. 20-22 days
21-Principle of tooth preparation in all except
A. Preserve tooth structure
B. Supra gingival margin ***
C. Resistance and retention
D. Structural durability
22-potency file
A. Push the file apically to remove debris or remove any block at apex***
B. Using circumferentially file to remove bio of lateral caries
C. Recapitulation with bleach
23-Pedo. Pt. has difficulty on communicating with people and in learning , he push the food instead of
swallowing it :
a. Down syndrome
b. Autism syndrome **************8
24-45 days baby during examination we find asymptomatic white, multiple nodules the mid of the palate.
what is you
diagnosis :
a. Epstein pearls*****************
b. Bohn nodules
c. Mucus retain cyst
d. herpetic
25-Pt. with bilateral swelling of mandible , profound swillingof maxilla , cheeks are stroked with lower eye
line depressed , x-ray radiolucent lesion with bone trabecular displacement of anterior teeth :
a. Fibrous dysplasia
b. Cherubism ***************8
c. Basal cell nevus
d. Giant cell granuloma
26-Polysulphide what needed
a. Pour 1 h***
b. Pour 12 h
c. It need pressure
d. Need a special instrument to temper it
27-What to prevent contamination restoration tech
a. Rubber dam***
b. Cotton roll
c. Salivary ejector
28-Child with physical disability and highly un cooperative with not present in the plan
a. Behavior shaping
b. Sending to home*************
c. Tight with board**
d. TSD
29-Major connector when to put in which phase
a. Examination
b. Diagnosis and ttt. plane ***
c. Tttt.
30-Loose enamel rods at the gingival floor of a class II amalgam cavity should be removed using
A. Straight chisel.
B. Hatchet
C. Gingival curette
D. Gingival marginal trimmer***
31-Loss of enamel surface due to acidic substance called
a. Abrasion
b. Erosion***
c. Attrition
32-For decrease periodontal problem most effective way in addition to antiseptic solution
a. Dental health education***
b. Through and remove of plaque
33-What mean dental (primer) function
a. Remove smear layer
copolymerize with the resinn ***
34-pedo pt. 2y old has intruded central incisor
a. Anterior Occlusal x-ray***
b. 2 bite wing
c. O.P.G
d. 2 bite wing +2 Occlusal x-ray
35-Lingual nerve is
a. Branch of facial nerve.
b. Branch of mandibular nerve.
c. Supply submandibular gland
d. Both b and c ***
36-Die is done to
A. Allow for better waxing ***
B. Adjust the errors in preparation
C. Make investment easier
D. Adjust correct finish line
37-In 1 cusp how many pin
A. 1***
B. 2
C. 3
D. 4
38-Maryland bridge indicate
A. 2 missing teeth
B. Young patient ***
C. When need translucency
D. When abutment is extensively restored
39-Fluoride application should be done
A. Every 1 year
B. Every 3 months
C. Every 6 months ***
D. Every 18 months
40-least corrosion resistance phase of amalgam is
A. Ag-Hg Gama 1
B. Cu-Ag
C. Sn-Hg(tin-mercury) Gama 2***
41-Pedo pt. , gray teeth , narrow at apex , easily detached
A. Amelogenesis
B. Dentinogenesiss ***
42-Patient with healthy gingiva , upon insertion of perio probe , it should stop at
A. Most coronal of junctional epithelium ***
B. 1/3 of coronal of junctional epithelium
C. 2/3 of coronal of junctional epithelium
D. Most apical point of junctional epithelium
43-What of these material are not noble material
a. Copper ***
b. Gold
c. Palladium
d. Platinum

Exam5

1 osseointgration between bone and dental implant


A connective tissue and bone
B fibroustissue and pdl
C titanium bone**********************
2 pt with xerostomia (though the img closely resembled abrasion) commonly seen
A attrition
B abrasion
C root caries***
3 (img of floating teeth) pt with mental confusion, renal calculi ,and high alkaline phosphatase
A hyperthyroidism
B hyperparathyroidism*******************
4 (img of swelling on the canine premolar region mandile bilaterally)
A exostosis****************************
B ranula
C sialadenitis
5 h file better than k file
A positive rake angle******************8
B negative rake angle
C more efficient with better cutting angle
6 to prevent polymerization shrinkage,
Place restn incrementally and increase curing time
Placr restn incrementally and start soft self start...************************
7 Restn perfect after one week becomes lighter
A underpolymerized*****************8
B overpolymerized with water absorption
8 Composite restn proximal with over hanging use
Segmental matrix and wedge*************************
Circumferential matrix and wedge
Toflmeir matrix
9 Overcontouring of restoratn enhances
A esthetics
B cleansing action
C plaque accumulation**********************
D compromised periodontium
10 Hand and foot keratosis, with severe loss of periodontium early years of age
A pappilon levefre syndrome*********************
B aggresive generalized periodontitis
11 Inc bleeding on probing gingiva, petechae on the palate,blood analysis all normal except 25,000 platelet
count
A lukemia
B hemophilia
C thrombocytopenia*****************************
12 Slightly bleeding increase cappilary fragility.....
A vit k deficiency******************
B hemophilia
13 Fracture surgery,the surgeon chose 2mm plate to fix..size of the drill hole to fix the plate to alveolar bone
A 1.5********************
B 2mm
C 2.5
14 After gingivectomy time taken for reepithelization of the gingival margins
A 5-14 days*************************
B 15-21 days
C 21-28 days
15 Minimum length of gp required to protect the post from leakage and reinfection..
A 3-4mm***************
B 4-6mm
16 After crownlengthening what is the length of biological width
A 1mm
B 3 mm****************
C 2mm interdentally
17 Cardio pt on warfarin undergoing a surgery
A pt 1-1.5
B pt. 2-2.5*******************
C ptt 1-1.5
D ptt 2- 2.5
18 Characteristics of enamel
A demineralizqtion followed by remineralization**************
B remineralization
C irre
19 2% xylocaine 1cc how much mg?
A 20 mg
B 36 mg********************
20 Pt with flabby tissue on ridge,what would u advice the pt
A impression will compensate the tissue****************************
B tissue provides stability
C surgical removal of tissue
21 Root perforation
A mta********************
B hydroxapetite crystals and bone graft
22 Pedo pt exo the root tips of mesial and distal fractured
A visualize and leve****************
B visualize and gently remove
23 Tooth surface _________ aftr continuous chewing of tobacco,hard food and vigourous brushing
A attrition
B abrasion***************
24 Substantivity is the property to release when required from the oral structures,is the property of which
material
A flouride
B chlorhexidine gluconate****************
25 Irrigant solution of choice for debridment
A naocl******************
B saline
26 Fracture in which only cortical bone is fractured
A green stick #***********************
B condylar neck #
C ramus and body#
27 Features of reversible pulpitis
A sensitivity on having sweets and drinking cold****************
B lingering pain on cold
28 Irreversible pulpitis can be caused due to
A increase intrapulpal pressure****************8
B stimulation of the dentinal tubules
29 Gmt used for
A gingival bevelling of enamel proximally***********************8
B remove the undermined enamel in class 11
30 Electric pulp test not valid in children
A a delta fibres late appearance********************
B c delta fibres late appearence
31 Pt with contact stomatitis would experience more of
A pain
B burning****************
C numbness
D itching
32 Pt to be given diazepam iv injection feels burning sensation at the site of injection
A large diameter of needle
B presence of methyl paraben
C presence of propylene glycol****************
D inj directly into the __ vein
33 Child caries and adult caries comparison
A dentine sclerosing as age advances*************************
B same rate progression
34 Question on malocclusion class 2
35 Glaze to prevent dehydration is done in
A composite
B gic********************
C porcelean
36 55 yr pt has root caries ideal resto preferred
A rmgic*****************
B gic
C dentine densitizing agent
37 Designing of occlusal rests before guiding planes
A cause improper size of occlusal seat****************
B improper size of guiding planes
38 Pt with antidepressants what is the dose of epinephrine,
A 0.02-0.04************
B 0.1 -0.2
39 Scarlet fever pt on continuous abs, notice white patches
A candidia albicans********************
B leukoplakia
C lichen planus
40 which phase does the differentiation of organs into external and internal takes place
A embryo******************
B Foetus
C morphodifferentiation
41 axioproximal surface preparation what is to be taken care of
A prepare with taper
B preserve the adjacent tooth structure*******************
41 bur used in proximal surface prepn
A radial
B fissure bur*****************8
42 preparation of axial surface for veneers
A torpedo diamond
B tapered fissure*****************
C torpedo
43 gingivectomy contraindicated in
A alveolar bone defect*******************
B supragingival bone defect
44 pit and fissure sealent failour is due to
A contamination during placement******************
B brushing soon after placement
C brushing before the placement
45 what should be done before pit and fissure application
A polish with polishing paste**************
B polish with polishing disc
46 something about physiologic occlusion
A esthetic and support
B function
C occlusal stability***********
Exam7

1- missing #12 ,,#11 protruded ,,#11 and #13 have lingual and proximal caries how to restore the missing
tooth ?
a-implant fixed prosthodontics
b-conventional fixed prosthodontics*****************
c-resin bonded fixed prosthodontics
d-veneer
2-fractured part of root of mandibular premolar during extraction how to remove it >>root tip elevator
3-sharp edges in bone after extraction >> bone file**********
4-PPE (it's protective personal equipment ) >> mask*****
5-proxy brush for embrasure type >>2
6-young patient after extraction heavy bleeding use
Most effective / powerful hemostat
a- Gelatin sponge
b- Bone wax
c- Oxidized cellulose**************
d-cotton soaked in epinephrine
7-gracey and universal differences but with different choices
8-difference between standardized and conventional gutta percha
9-squamous cell carcinoma in tongue prognosis:
a-good prognosis with low recurrence
b-good prognosis with high recurrence
c-poor prognosis with low recurrence
d-poor prognosis with high recurrence**************8
10-most common used blade in dental sugeries >> blade no.15
11-minimum crown/root ratio >>1:1
12-GG size 1 >> 50
13-conditioning GI >> 10 % polyacrylic acid for 10 sec
14-chroma >> saturation
15-bleaching in clinic :
a-38% sodium perborate
b-38% hydrogen peroxide************
c-10%hydrochloric acid
d- 5-22 % carbamide peroxide
16-multiple impactions ,,supernumerary teeth >> i don't remember choices i chose cleidocranial dystostosis
17-Gates Glidden:
a. Has numbers 6-9
b. For coronal preparation*******************
c. Breaks if bind inside the canal
D. for root preparation
18-hypertensive pt 210/100 how to manage the case :
a-LA without epinephrine
b-it's just the fear from dental chair
c-delay to another day and refer to physician*************8
d-...
19-Types of steel used in many dental instruments:
a- stainless steel and tungsten steel
b- stainless steel and cobalt steel
c- stainless steel and carbon steel***************
d- carbon steel and tungsten steel
20-patient came to hospital with gun shot ,,the surgeon will make fixation by :
a- christian's technique ***************
b- keen's technique
c-....
d-...
21-pic with multiple erosion on the patient's teeth ..he has GERD .. will make full crowns what's the best
cement :
a-GI cement
b-zn phosphate
c-resin***************
d-ZOE
22-After class II amalgum restoration in a young patient ,he came with a fractured isthmus :
A. high filling*************
B. Over flare of the cavosurface angle.
C. improper amalgam mix
23-Pt. With upper complete denture for many years came with denture not firmly attached to palate .. And
found multiple soft lesions on palate:
a. epulis fissuratum
b. hypertrophic frenum
c. denture stomatitis**************
d.palate chelitis
24-trauma to tooth after 19 month no difference and no evidence of any pathosis >> wait and see
25-case about sharp momentary pain no pain on biting >> reversible pulpitis
26- conc of chlorhexidine mouth wash >> 0.12%
27-When xray cone is not properly pointed what happens?
A. Cone cut*************
B. Overlap
C. Elongation
D. Shortening
28-x-ray with resorbed primary upper cantral mesiodens and permanent upper central ..how to manage the
case :
a-extract upper primary central
b-extract upper primary central and supernumerary tooth**************
c-leave and follow up
29-AH plus advantages over AH 26 >> no release of formaldehyde
30-pain on hot drinks no pain on biting the pain lasts for 10 minutes >> irreversible pulpitis
31-biological width after crown lengthening >> 3mm
32-In placement of rubber dam:
a) Four points of contact 2 buccally and 2 lingually without rocking.***************
b) 4 points of contact 2 buccally and 2 lingually above the height of contour.
c) 4 points of contact 2 mesially and 2 distally.
d) 2 points one buccally and one lingually
33-question about cardiac patient need prophylaxtic antibiotic and penicillin allergy there were many
antibiotics options with different doses clarithromycin ,, azithromycin ,,
Clindamycin used in case of penicillin allergy****************
34-We give prophylactic antibiotic coverage to which of the following patients before dental treatment:
A- recent bypass surgery
B-myocardial infarction
C-Patient with Prosthetic valve***************8
35-epithelialization occur after >> 5-14 days
36-The bacteria communicate with each other by :
a. courum sensing **************8
b. courum signaling
c. courum transfer
37-At the beginning of the operation day in the clinic u should use water air spray for 3 min. for which Micro
organism >> Pseudomonas aeruginosa
38-Most appropriate one to describe pulp pathosis or (degeneration)..
A. Diapdesis of leukocytes************
B.irreversible pulpitis
C.diseased pulp
39-System-B obturation technique is?
A) cold GP. Condensation
b) warm GP. Condensation
c) continuous condensation******************
40-class 3 kennedy classification def
41-soft tissue prognathism found in:
1 class I maxillary mandibular relation
2class II division 1*****************
3 class II division 2
42-Why do we clean instrument before sterilization?
a-Less time sterilization
b-Preserve instrument integrity***
c-Kill protein
d-Kill spores
43-most common of these : cleft lip ...cleft palate .. cleft lip and palate*************
44-water fluoridation >>1 ppm
45-Function of water stream device?
a. plaque removal
b. prevention of plaque
c. dilution of bacterial toxin**
46-Edentulous pt.classII kenndy classification 2nd premolar used as abutment when we serving we found
mesial undercut what is the proper clasp used:
a-wroughtwire with round cross section.***
b-wrought wire with half round cross section.
c-cast clasp with round cross section.
d- cast clasp with half cross section.
47-the component in RPD that opposes the force of the retention arm does:
a. reciprocation.****
b. Support.
c. Stabilization.
48-2wall defect in perio what is the best graft to treat this defect
A. Cortical freeze dried bone allo graft
B. Cancellous freeze dried bone allograft*************
49-in rotary we use water device to :
a-avoid dentin burn************
b- flush dentin debris
c-have a better view
50-difference between acute apical abscess and chronic apical abscess >> sinus tract
51-gag reflex when we take lower molar periapical film :
a- because of lingual nerve*******************
b- gingiva and buccal mucosa
52- what of the following can cause injury to occulomotor nerve :
a-pupillary constriction
b-something like trauma to the eye
i don't remember the other choices
53-root resorption >> magnitude
54-Surfactants used to:
Increase surface energy**************
Decrease surface energy
55-caries in proximal surface what type of x-ray we use >>bitewing
56-condylar fracture what type of x-ray >> reverse town
57-child ingested lethal dose of fluoride what's the dose and when will he die ..he didn't give any information
other than that :5-10mg/kg
58-best type of bone for implant >> type 2
59-pedo case about apexification
60-Clinical remounting is :-
1-usually require face bow record
2-usually require split cast
3-usually require centric relation record***************
4-indicating in gross occlusion error

Exaam8
1. Bone removal during alveoplasty
A. Bone file for bone smoothening***************
B. Osteotome
C. Rongeur for bone cutting
2. Maximim amount of Anesthesia 80kg patient lidocaine 2% + epinephrine 1:100,000
Answer: 7*80= 560mg
3. Complex in 18 year old healthy patient: purple
4. Patient going for ortho consultation, no crowding, history of trauma to anterior teeth
Which class?
1. I
2. II div 1 ***
3. II div 2
4. Class 3
5. Fatal fluoride toxicity for a 3 year old with time 5-10mg/kg
6. Amount fluoride 5 year old, 0.5ppm water fluoridation
A. 0.25 ***
B. 0.5
C. 1
D. 1.25
7. Color saturation: chroma
8. Hold epulis fissuratum with: Allis forceps
9. What is maryland bridge?
Answer: Resin bonded
10. Child with lower canine infection needs extraction -no crowding-, what do you do?
Answer: extract two canines without space maintainer
11. Tmj innervation: auriculotemporal nerve
12. Distance between 2 implants: 3mm
13. Crown root ratio if crown:10mm and root:15mm?
1:1
1:2
2:3***
2:1
14. 3 yr old with 5mm intrusion of upper incisor what do you do?
1. Wait and see if erupts ***
2. Extract + space maintainer
3. Crown lengthening
4. Reposition manually and splint
15. Type of ortho force for teeth with compromised periodontium: light
16. In primary teeth, before putting filling you do: Proper cleaning and drying*****
17. How to get rid of blood, suction fluid waste: drain connected to sewage
18. Cleft lip result of:
Malunion of medial nasal and maxillary process
19. Maxillary growth:
deposition at Tuberosity + resorption at nasal + downward and forward movement
20. Best for disc perforation: arthrography
21. Child fell on chin with both condyles broken but all is normal (no malocclusion, all movements are fine)
treatment:
No treatment
22. Suspect fracture right condyle, xray? reverse town
23. Patient pain on biting (relief when open) related to tooth with big amalgam restoration, confirm diagnosis
by? Tooth slooth
24. Patient pain on biting with deep pocket related to one root
Suspect? Vertical fracture
25. Patient dull pain related to tooth with endo since 3 yrs ago. Short endo+ radiolucency and restoration
leakage.. reason for failure?
Short endo + inadequate filling
26. Child came with pain and fatigue + multiple vesicles on hard palate (last year he had the same and went
away after one week). Diagnosis?
Herpes simplex (recurrent)
27. Streptococcus mutans in? Deep pit and fissure
28. Angle of instrument with tooth? 45-90
29. Advantage GIC in carious mouth: fluoride release
30. Restoration needs glaze to prevent dehydration?
A. GIC ***
B. Porcelain
C. Composite
31. Adult Patient with broken tooth between middle and apical third, managment?
Endo for coronal part
32. Patient adviced to brush her teeth frequently to prevent calculus, why? To break plaque layer
33. Disinfect gutta percha by: sodium hypochlorite
34. Gutta percha formed of?
70% Zno Eug and 20% gutta percha
35. Difference btwn standerdized and conventional gutta percha (forgot answers)
36. Question about the strip that changes color when autoclave, what does it mean?
A. Object subjected to steam heat******************
B. Object sterilized
37. Why do we clean instruments before sterilization?
A. Less time sterilization
B. Preserve instrument integrity
C. Kill porion proteins***************
D. Kill spores
38. What type of disease you can work with at clinic?
A. Conjuctivitis
B. Chicken pox
C. Hepatitis C***************
D. Something like chicken pox
39. Diabetic with pain and abscess and allergy to penicillin:
A. Start working and prescribe clarithramycin
B. Prescribe clarithramycin then start work**************
C and D had penicillin
40. Anesthesia with less pain
A. Topical
B. Stretch tissue
C. Inject slowly
D. Needle gauge more than 25
And there were options to choose from, i choose A,B,C
41. What anesthesia has most vasoconstrictor action?
A. Cocaine8******************
B. Tetracaine
C. Procaine
D. Articaine
42. Action of smear layer in pulp protection?
A. Dsnt allow toxins reach pulp*****************8
B. Less effect of etching on pulp
43. Old patient with sharp severe pain on trigger and doesn't cross midline: trigeminal neuralgia
44. Which tooth has root that is mostly pushed into sinus when extraction?
Maxillary first molar
45. Which fibers are teared first when extraction?
A. Transceptal
B. Circular
D. Dentoalveolar ******************
E. Cementoalveolar
46. Which smooth surface is mostly affected with high caries patient?
A. Labial maxillary incisors
B. Palatal maxillary incicors
C. Buccal maxillary molars **************
D. Palatal maxillary molars
47. Tongue tie treated when? When limitation of tongue movement
48. Positive rake
A. K file********************
B. K reamer
C. ProFile
D. Protaper
49. Case about patient with porcelain teeth in upper denture and now needs lower denture, what type of teeth
we use? I chose porcelain (there were other options like porcelain+gold or acrylic)
50. Why do gingivectomy? To remove pseudo pocket
51. Why use low speed with pedo?
Less pulp exposure
52. Baby with Lesion anterior maxilla: congenital epulis
53. When to do re endo if exposed to oral fluids?
6 days (there were other options)
54. Something with epoxy resin, what type of impression?
A. Polyether
B. Polysulfide
C. Additional silicone
55. Most difficult group of teeth to anesthesize? Mandibular molars
56. Centric relation? Bone to bone
57. Bleeding from nose with doubt of cerebrospinal fluid?
Glucose oxidase formation
58. Patient with esthetic concern having dark central bcz of trauma treatment?
A. Crown
B. Veneer
C. endo
D. Endo+internal bleaching*************
59. Most size suture material in dentistry? 3/0
60. Patient came for restorative work with Blood Pressure 200/160 treatment
A. Anti stress
B. Anesthesia without epinephrine
C. Nothing, it's just chair stress
D. Reschedule and send him for physician consult ***
61. Property of material that doesn't distort with tensile stress
A. Ductility***************
B. Malliability
C. Solubility
62. Child multiple white spots, inflammed gingiva, hight plaque, probing depth 2-4mm
Periodontal diagnosis?
A. Gingivitis ***
B. Jeuvenile periodontitis
C. High caries index
63. Prophylaxis for patient with bacterial endocarditis:
A. Ortho band placement***********
B. Routine dental cleaning
C. NON ligamentary injection
D. Suture removal
64. Amalgam copper: 13%
65. Median rhomboid glossitis with patient wearing denture cause? Candida
66. Type of approved probe for exploring pit and fissure sealant?
Blunt
67. Most common type of occlusion comfortable for patient and easy to fabricate?
A. Mutually protected****************
B. Unilateral
C. Bilateral balanced
68. When xray cone is not properly pointed what happens?
A. Cone cut****************
B. Overlap
C. Elongation
D. Shortening
69. Patient low caries worried about brown discoloration of premolar (picture) diagnosis? Stained groove
70. Interdental brush with? Type II embrasure
71. Endo central minimal with mesial caries. Restoration?
A. Composite
B. Fiberpost + mesial composite
C. Cast post/crown*************
72. Name of injury caused by normal/abnormal occlusion on a periodontally compromised tooth?
A. Fremitus
B. Primary occlusal trauma
C. Secondary occlusal trauma*************
73. When to decide Composite shade?
A. Before rubber dam placement*************
B. After cavity
C. After bonding
74. Best way to know pulp status and integrity?
A. Cold test**************
B. Electric test
C. Anesthesia
D. Percussion
75. Case picture lesion, patient all fine but he had trauma to the area (he bite it traumatically). Diagnosis?
A. Fibroma********************
B. Papilloma
C. Hematoma
76. Case 12 yr old child swelling in area 44 45 (missing 45) xray radiolucency around crown 45 +
radioopaque specks. Diagnosis
A. Dentigerous cyst
There were other options i dnt remember
77. When sharpening curette angle? 100-110
78. Cast post and core try in
A. Seat With pressure
B. Seat Without pressure************
C. Rotate
79. Irreversible pulpitis bcz of?
Congestion of blood vessels in pulp
80. After gingivectomy, surface epithelialization occurs after?5-14 days
82. Compmer is? (Composite resin + polyacid i think)
83. Porous denture managemet
A. Reline
B. Rebase**************
C. Remake
83 teeth present at the gae of 10
All permanent incisors first molars lower canine and upper first premolars
Exam9
1) Concentration of CHX in mouth washes ? 0.12%
2)
2) Pt. got gun shot , question about graft used for condyle ? Costocondral
3)
3) Nerve most likely to get injured in case of injury to condyle ? Auricotemporal nerve
4)
4) Fluoride in water ? 1 ppm
5) female taking water 8 ppm , how much is the supplement ? Zero
6) Implant made of what ? Titanium
7) Focal infection theory, is systemic infection caused by ?
A- Systemic Infection
B- Local Infection**************
C- External Trauma
8) Bacteria which can invade epithelium?
A- Porphyromonas Gingivalis***********
B- Fusobacterium
C- Streptococcus mutans
D- Capnocytophaga
9) Pt got root remnants with 3x4 cm radicular cyst, Management ?
A- Root canal therapy
B- Extraction of roots with curretage through the root socket
C- Extraction & curretage trans buccally **********
10) Feature makes lower 1st primary molar difficult to handle?
A- Narrow occlusal table
B- Occlusal transverse ridge
C- Buccolingual wide contact
D- Mesiobuccal bulge***************
11) Meaning of immediate implant ? Placement of implant at time of extraction
12) Treatment of mucocele ? Excision
13) Child 3 yrs old with oral candidiasis ?
A- Nystatin oral suspension************
B- Fluconazole systemic
C- Ketoconazole systemic
14) Recurrent herpetic gingivostomatitis ?
A- 2% acyclovir ointment**
B- 4% acyclovir
C- Assure pt it's self limited
15) Bacteria causing Acute osteomyleitis ? Staph aureus
16) Case about female pt wearing denture for 3 yrs, she got asymptomatic general redness in the
denture bearing area , what is the diagnosis ?
A- Denture stomatitis*********
B- Papillary hyperplasia
17) Long case Pt with denture got ulcers in vestibule ? Epulis Fissuratum
18) Case & pic, of pt had ortho treatment, she got white spot on ( 12 ), they asked LESS option to be
done when the concern is only esthetic?
A- Composite Veneer
B- Porcelain Veneer
C- Conservative composite restoration************
D- Full ceramic crown
19) Difference between Gracey & Universal currette ? ( same as files )
20) In what aspect is acrylic teeth better than porcelain teeth in RPD ? Acrylic teeth bonds better to
denture base
21) During crown preparation the finish line was subgingival what we should use ?
A- retraction cord*************
B- electrocautery
C- gingivectomy
22) Special sensation of posterior 1/3 of tongue by which nerve ? Glossopharyngeal nerve
23) Pterygopalatine ganglion related to which gland ? Lacrimal
24) Pt got acute asthmatic attack, management ?
A- Epinephrine, subcutaneous 1:100
B- Epinephrine, subcutaneous 1:1000************
C- Epinephrine, subcutaneous 1:10000
D- Epinephrine, subcutaneous 1:100000
25- Function of lentilo spiral in fabrication of post & core?
Coating separating medium intra canal*************8
26- When is initial survey done ?
A- During denture insertion
B- Before taking impression
C- On working cast
D- On diagnostic cast**************
27- Female 9 yrs old, most susceptible area for caries ?
A- interproximal of premolar
B- occlusal of permanent molar**
C- buccal of canine
D- mesial of central incisor
28- PFM incisal reduction ? 2 mm bucal and lingual,,,1-1.5mm
29- File 20 , what's the tip width ? 0.2 mm
30- 2% taper endo file, what does that meam ?
A- From Tip to D16*********8
B- difference with rotary files
C- increase taper of next file
D- 0.2 mm more than other files
31- Dry socket , occurs ?
A- immediately
B- 3 to 4 days*************
C- A week
D- within hours
32- Impacted lower canine , which x-ray ? Lower Occlusal
33- Best x-ray for implants ? CT
34- Pt requires AB prophylaxis, allergic to pencillin ? Clindamycin 600 mg one hour before
procedure
35- Acute Periodontitis ?
A- Invasion of PMN's & edema**************
B- Chronic accumulation of surroundings
C- Sinus tract
36- Herpetic gingivostomatitis characterized by ?
A- Not painful
B- Not contagious
C- Spherical grey vesicles*************
D- Pseudomembrane
37- Preventative Resin Restoration done to prevent ?
A- Initial caries
B- Caries progression***************
38- Pt got non cavitated white carious lesion, what's the time intervals for bitewing?
A- 12-18 months*****************
B- 18-24 months
C- 24- 30 months
39- Streptococcus mostly where ? Pits & Fissures
40- Method to prevent caries in children
A- Preventive resin restoration
B- P&F sealant*************
41- Hyperventilation management ? Breathe in bag
42- Pulp pain by :
A- Merkel cells
B- Free nerve endings*************
C- ( some other things first time i see !)
43- Restorative material for xerostomia ? GIC
44- Smear layer role in protecting the pulp ? Reduce diffusion of toxins to pulp
45- Long Case of pt got solitary cyst , what is the treatment? Curretage & closure
46- Female need extraction, she got hemophilia B , which factor is affected ? Factor IX
47- Dentinogenesis Imperfecta occur in which stage ? Histodifferentiation
48- Same question but enamel hypoplasia ? In apposition stage
49- Overcontoured crown enhances ?
A- tensile strength
B- plaque accumulation****************
50- 1st step in removing the rubberdam ? Cut interdental septal
51- Base under restoration ? High modulus of elasticity
52- repair of porcelain by ?
A- 37% phosphoric acid
B- hydrofluoric acid 4-10%****************
C- hydrofluoric acid 30%
53- We wait 6 months after perio surgery for ?
A- Maturation of pdl***************
B- Complete epithelialization
54- Vasoconstriction in LA ? Reduce toxicity
55- Proxy brush used for ? Embrassure type II
56- Crown with short margin, after doing the best to fix it .. What is the best thing to do ?
A- Seal with cement
B- Repeat crown**
57- Appliance for thumb sucking which phase ?
A- Primary dentition
B- Early mixed****************
C- Late mixed
D- Permanent
58- Long Case describing lesion radiolucent around root of vital teeth with radiopaque border ?
Simple bone cyst
59- Pt got discomfort in premolar & in x-ray sudden disappearance of canal ? Bifurcation of canal
60- Child got stains in some teeth , 36 got one point where the explorer got stuck .. Treatment ?
A- fluoride
B- PRR**************8
C- P&F sealant
61- DIAGNOdent ?
A- Qualitative
B- Quantitative****************
C- both
62- About rubberdam clasp, best for stability ? 4 points of contact
63- Growth at condyle ?
A- appositional
B- apposition + intramembranous
C- appositional + endochondral**************
64- Pt got intrusion after trauma, x ray shows progressive root resorption, diagnosis:
A- Replacement resorption
B- Surface Resorption
C- Cervical resorption
D- Inflammatory resorption***************
65- Pt with upper CD & lower 6 anterior teeth , what is seen ? Resorption in max anterior
66- pt complain of pain, in examination found large carious lesion, which x-ray ? Periapical
67- Case of pain & parasthesia with moth eaten in x-ray ? Acute osteomyelitis
68- Case about child got fever with ulceration & many symptoms i forgot & asked about condition?
A- Acute myeloid leukemia
B- Acute lymphocytic leukemia**
C- Chronic lymphocytic leukemia
69- how long it takes for autoclave cycle ? 30 mins

71- Nerve supply of palate in anterior region ? Nasopalatine nerve


72- 7 yrs old child with complicated trauma , large pulp exposure came after 48 hrs, management ?
A- RCT
B- Extraction
C- Full pulpotomy**
D- Partial pulpotomy
73- Pt with ortho treatment got resorption in middle of root, treatment? Ca(OH) in site of resorption
74- Long Case of dentigerous cyst & ask about treatment ? enucleation with follow up
75- Picture of cast & ask about the class of occlusion?
76- question of odd ratio of cancer for smoking & non smoking ? 13.5 ( same as files)
77- Pic of broken upper CD from frenal area to post dam ( midline fracture of Denture) after 1 yr ask
about reason ?
A- Thick frenum attachment
B- unbalanced occlusal forces***************
C- Thin denture
78- Pt with denture complains of poor retention , when u press on palate u see bubbles in posterior
extension due to :
A- Over extention
B- Over postdamming
C- Under postdamming**************
D- Under extention of postdam
79- The same question in file about the swing lock denture
80- Role of anesthesia in reducing saliva?
A- Blocking cholinergic ends
B- Reduction of anxiety **************
C- Blocking Salivary glands
81- Characteristic of usage of electrosurgery ?
A- Light pressure with deft strokes********************
82-Clicking of new denture ? Increased VDO
83- Retention of restoration ?
A- Prevent dislodgment apical direction
B- increase tensile strength
C- prevent dislodgment in the direction of pathway of insertion**
84- pt have missing 15,16,18,25,26,27,28 have good bone support & caries free with no mobility ,
asking about type of clasp on 17 for RPD ?
A- akers******************
B- ring
C- multiple
D- compound
85- Prion ? High resistance to heat
86- Unacceptable way to grasp hand instrument ? Pen grasp
87- Most common for discoloration of non vital tooth ?
A- Drinking coffee
B- hemorrhage of pulp***********
88- Movement produced by removable appliance ? Tipping
89- Canal most likely to be missed during RCT of lower central incisor ?
A- Buccal
B- Mesial
C- Lingual***************
D- Distal
90- Ortho force on weak periodontal teeth ? Light force
91- Cell responsible for bone resorption during ortho treament ? Osteoclast
92- system B?
A- warm GP condensation **********
B- cold condensation
C- chemical plasticized
D- injectable GP
93- Least prognosis of endo involved tooth ?
A- Necrotic pulp without periapical lesion
B- Previously treated with good RCT
C- Necrotic pulp with periapical lesion **
94- Airborne infections mostly by ? Aerosols
95- Avulsed tooth kept in ? Milk ( There was no HBSS in choices )

Exam10

1- What is the etiology for posterior open bite in children during the eruption of lateral incisor
a.Functional shift
b.Imbalance between tongue and cheek
c.Improper swallowing*****************
d.Improper chewing
2- . Patient received blow on face, horizontal middle third fracture and symptomatic. No response to pulp test
and no radiographic change. Management-
- rct coronal*********************
- Extract apical part
- Extract both parts
- No treatment for now
3- Lower right second molar treated by temporary Rct 6 months ago. Patient has pain, bone loss, normal
periodontal depth. Reason?
-vertical root fracture
- Missed lateral canal
- Missed root canal
-Leakage*****************
4- After crown lengthening you wait for 6 months before placing a crown for
Junctional epithelium formation****************8
Maturation of periodontium
cementum formation
5- Pterygopalatine ganglion related to which gland
Submandibular
Lacrimal******************
Sublingual
Parotid
6- Early treatment is orthodontics in cases
midline shift not in functional shift
slight cross bite*********************
when primary molar is slightly distal
slight crowding in anterior region
7- Suspecting that right condyle is shifted mesially , which x-ray
a. Occipitomental
b. Reverse towns ********************8
c. Lateral oblique with 30 degrees
d. PA for mandible
8-True about odontoblastic layer?
a. Filled with odontoblastic processes*****************
b. Charectarised by cell-to-cell gap junction
c. Charectarised by cell-to-cell tight junction
d. Have higher concentration of cells?
9- What is the gauge of the needle used in fine needle aspiration of Keratocyst?
a. 19******************
b. 21
c. 23
.10- The forces action through a FPD on to the abutment tooth should be directed : 1. As far as possible at
right angles to the long axes of the teeth.
2. Parallel to the long axes of the teeth.
3. By decreasing the facio-lingual dimension of the pontic
4. By decreasing the Mesio-lingual dimension of the pontic.
5. In a mesial direction , teeth nearer the midline will offer additional support.
A. 1+3+4.
B. 1+2+5.
C. 1+4+5.
D. 2+3. ***************************
E. 2+4.
F. 2+5.
11- What is the etiology for posterior open bite in children during the eruption of lateral incisor
a.Functional shift
b.Imbalance between tongue and cheek
c.Improper swallowing*************************
d.Improper chewing
12- A patient visits the clinic to restore a tooth. During examination you found
white lesion with elevations filled with pus on the cheek. What is your treatment
A. Refer to oral surgeon for biopsy **************************
B. Give local anaesthetic for 2 weeks and recall
C. Recall for follow up after 6 weeks
D.
13- Which is the technique that will anesthetize both hard and soft tissues of the lower posterior teeth region
in one injection
A. Gow gates**********************
B. Inferior alveolar nerve block
C. PSA
14- A patient comes with mobile incisors after trauma. The fractured alveolar portion moves as one unit.
What will see in an iopa
A. Gap between root tip and bone **********************
B. Lost trabacule
C.Lost pdl space
15- A patient gives history of previous intrusion of incisors. Iopa shows loss of pdl space and progressive
root resorption. What is your conclusion
A . Inflammatory resorption *****************8
B. Repositioning resorption
C. Surface resorption
D. Cervical root surface resorption
16- Pateint complaining of sever spontaneous pain related to maxillary 16 ..........responds to vatality test and
on percussion no pain.
A. Symtomatic irreversible pulpities*******************
B. Asymtomatic irriversible pulpities
C. Revesible pulpities
D. Acute apical perodontitis
17- What best defines pain perception?
a. unpleasant reaction to obnoxious stimul************************i
b. different responses of different individuals to same stimulus
c. different interpretation of a particular stimulus
18- Multiple radiooppaque radiolucent lesions in both jaws seen on OPG, painless, doesnt expand bone, no
extraoral or clinical findings, vital teeth?
Florid osseous dysplasia************************
Polyostotic fibrous dysplasia
Peripheral ossifying fibroma
Multiples osteomas
19- A finger spring was to be used to correct a mild crossbite of upper later incisor. In which of the following
situations is it preferabble to correct a crossbite with a finger spring?
Lingual tilted lateral incisor , 5% overbite**********************
Lingual tilted lateral incisor , 50% overbite
Labially tilted lateral incisor , 5% overbite
Labially tilted lateral incisor , 50% overbite
20-Delcaficiation in case of fixed orthodontic appliances occurs most commonly on which surface of tooth?
Lingual, Interproximal, just behind brackets,
just around brackets************
21- Motor supply of scalp? Temporal branch of facial nerve***********
, Supratrochlear,
Supraorbital,
Auriculotemporal nerve (V3).
22-11 or 12 year old girl suffered a trauma on the fromt tooth. Mildly Discolored tooth, Root fracture with
radiographic wide space between the two segments. (Nothing was mentioned about mobility) Treatment?
Extraction*******************
RCT of coronal part
RCT of apical part
23- Case with blow to ride side of face, midfacial fracture, subconjunctival hemmorhage, hematoma(or was
it bruising) in maxillary buccal vestibule, difficulty in opening mouth?
Lefort 1,
Lefort 2,
LEfort 3,
Zygomaticomaxillary complex #*****************
24-Type of forceps used for extarction of maxillary premolars?
No. 1 universal***********
, No 88 right and left paired ,
Maxillary No 205,
25- Periodontal flap in which epithelial lining of periodontal pocket gets converted into attached gingiva?
Modified Widman flap, Apically positioned flap**************, Undisplaced flap
26-Curing time for P&F sealant?
10-20 secs***************
20-30 secs
40-50 secs
60-70 secs
27 Endo treated Mandibular central incisor with narrow canal has tooth structure only 1 mm above gingival
margin. Whci his the best post endo restoration?
Carbon fibre post with composite core*************************
prebfabricated ss post with composite core
amalgam core
cast post core
28- Width of Ginigval floor of Proximal box of amalgam? 1.5mm*************
1mm, 0.5mm, 2mm
29- Bevel ised in small class 3 composite>
Full bevel, Long bevel*************
, Short bevel,
Hollow Ground (concave) Bevel
30- In Pts with Diabetes mellitus, infections are due to defect in ?
PMNS, Lymphocytes, Macrophages************, Immunoglobulins
31- A 21 year old man comes to the dental clinic. You notice he has plaque amd calculus accumulation. A
SCaler with elliptical motion was used in this patient for scaling. Which type of scaler ?
Piezoelectric, MAgnetostrictive************, Sonic, Ultrasonic
32- Theoretically which Xray technique gives exact length of root canal ?
a. Bisecting angle
b. PAralleling technique************
33- Pulpitis from composite restoration usually occurs because of which content?
Polymer,
Monomer***************
34- Which salivary gland is functionally more important in swallowing?
Palatine
submandibualr*****************
Anterior lingual
Von ebner
35- Advantages of ah plus over ah26?
Formaldehyde release reduced *****************
Eugenol was added
Sealing ability is better
35- a 34yr old patient had pain at his lower molar 2weeks before. Nw that area is inflamed and
swollen(diffuse) , the tooth is grade 3 mobile..what is your management.
a) give antibiotics and refer to oral surgeon
b) emergency opening of the tooth, allow pus discharge. Prescribe antibiotics
c) extract the tooth+ prescribe antibiotics*****************
36-while placing a self threaded pin the pin accidently goes into the pulp chamber. What is your
management
a) place calciumhydroxide and proceed the treatment
b) cover the tip of the pin with calcium hyroxide, replace it and proceed**************
c) do RCT
d) extract the teeth
37-some description about accessory canals
a) accessory canals are commonly seen at bifurcation of maxillary molars
b) accessory canals are commonly seen at bifurcation of maxillary and mandibular canals**************
c) accessory canals are seldom seen at bifurcation of molars
d) accessory canals are seen only at apical region of molars (something like that, not remembering)
38- sealer extruded from the apex...
a) zno is more irritating than ah plus sealers
b) ah plus is more irritating than zno
c) endofill is more irritating than ah plus
d) zno and ah plus are more irritating than endofill*******************
39-most common complication of calciumhydroxide pulpotomy
a) canal calcification
b) internal resorption********************
40-)Primary sign of gingival infection
.change in consistency
. Bleeding on probing
. Change in color **************
41- which impaction mostly needs sectioning of teeth
. Veritcal
. Mesioangular
. Horizontal *****************
42- Pt seeking ortho in examination you found that she has amelogenesis imperfecta
What type of brackets you should use?invisalgn

42.nerve cell dont repair once dead


43 split dam technique used in fixed partial denture field
Exam11
14- Whats the ratio :
Oral cancer Control
Smoker 90 10
Non smoker 40 60
a- 6
b- 2.355
c- 3.43
d- 13.5********************
16- an old patient and has skeletal disorder by using x ray found enlargement in . Whats the type of x ray
should be more accurate :
a- Cephalometric****************
b- Reverse town
c- Panoramic x ray
d-
20- whats the mainly factors that effect in healing
a- intraossoius healing
b- connective tissue healing
c- inflammatory cells *****************
d- inflammatory fluids
21- How can be standraized for intracanal instrument
a- length of the blade
diametr of tip of the ******************
c- length of the hand
d- size of the hand
22- child have truma and by x ray you found fracture in mid root ( permanent tooth)
a- leave without any treatment and allow the tissue healing ***************
b- make RCT immeadiatly
c- using direct pulp capping and follow up
23- patient has missing #45 need restoration
a- Maryland bridge
b- FPD***************
c- implant
25- flux used for
a- prevent oxygen from contacting alloy
b- dissolve oxide
c- a and b**************
26- patient has upper #6 RCT with small MOD caries, best treatment
a- MOD gold inlay
b- MOD gold onlay
c- gold crown***************

28- case
patient diabetic , upper #5 have amalgam MOD, and have mobility grade II, calculus quit, and want to make
artificial for missing tooth
a- FPD with tooth supported
b- extraction and make RPD**********************
c- make occlusal rest and make RPD
d- remove it and make implant
e- resin bounded tooth supported
29- whats the best material under composite
a- caoh
b- GIC*********************
c- zinc polycarboxilate
d- zinc

35- redness on residual ridge at denture wearing patient


a- denture stomatitis ********************
b- epulis fissurtum
38- 9 yrs old patient with bilateral swellimg in the mandible ( multilucular)
a- ostiomilytis
b- ameloblastoma
c- cherubism***********************
42- soft tissue protrusion in
a- class I
b- class II
c- class III
d- class II mod 1*******************
44 - systemic antifungal drug
a- nystatin topical
b- fluconazole***************
c- amphotacin
45- infective indocarditis bacteria
a- streptococcas virdens ***************
b- strept. Salivareis
c- A & B
46- muscle for control cleft palate
a- levator .. palatine *********************
b- tensor ..palatine
47- patient has periodontal disease which in lower #6 has furcation grade III, mobility grade II, moderate
bone loss, short root , whats the prognosis
a- fair
b- moderate
c- poor******************
57. child had trauma in the upper central and become intruded with loss of superficial layer of epithelium this
is
1. concussion and subluxation
2 laceration and subluxation
3.laceration and luxation*****************

61. Teeth with RCT and you want to use post, which post is the least cause to tooth fracture : 1. Ready made
post. 2. Casted post. 3. Fiber post***************. 4. Prefabricated post.
65-Neonatal teeth;
a- before birth
b- 0-30 month****************
67- Orthognathic surgery , you plan to use 2mm screw , the drill size :
a- 1 mm
b- 1.5 mm***************
c- 2 mm
d- 2.5 mm
69- In case of resorption of alveolar ridge, the percussion when do crestal incision posteriorly not to damage
:
a- lingual nerve**********************
b- buccal nerve
c- mylohyoid nerve
d- alveolar nerve
71- The most ideal film thickness cement :
a- zinc phosphate *****************
b- zinc polycarboxilate
c- GIC
d- Resin cement .

Exam12
1. 18 years old pt. the bacterial complex present in his mouth
is :
A. Red complex
B. Green complex
C. Purple complex **
2. Pt allergic to both amide nd ester.GA cnt b done.absolute need of LA..we use..
diphenhydramine***********8
3. LA most common complication arises from
a.Drug to drug b.Vasoconstrictor c.LA drug d.Stress
4. long case about dentigerous cyst
5. patient needs LA for surgery . weight 80 kg. 1:100000 la max dose hw much
a. 320
b. 400
c. 560**
6. Indirect sequelae of long term wearing of denture
a. Ridge resorption
b. Masticatory muscle atrophy ***
7. Mucocele treatment
a. Excision ***
b. Cauterization
c. Electrosurgrey
8. Kid has catch in occlusal surface .no caries . treatment
a.Fluride therapy b. Pit and fissure sealant *************
9. Long case with a photograph . past history sun exposure
Keratoacanthoma
10. Blood supply to upper first premolar..posterior superior alveolar artery
11. Persistant bacteria in root canal
e.fecalis
12.. microbial flora present in sinusitis
a.primarily aerobes
b. primarily aneorobes *************
c. 80 % aerobes 20% aneorobes
13. forceps for holding epulis fisseratum ( ds qstn was asked twice in 2 ways)
a. Allis forceps
14. GG 1 # tip diameter same as which file size
a. 20 ** b. 40 c. 30
15. heamophilia A what is deficient
A. prothrombin
b. factor VIII***
16. what happens in condyle
A. apposition and endochondral displacement****************
b. resorption and endochondral displacement
c. apposition and intramembraneous displacement
17. condyle displacement which xray its reverse town
18. favoured relationship in class III case for FPD
unilateral balance***********
19. smallest diameter in root canal
Apical constriction************
20. rigid rpd design which framework
Cobalt chromium
21. Spedding principle used for what to choose stainless steel crowns
22. best definition for tertiary dentin
A. calcified dentin tubules
b. odontoblastic process
c. irregular reactive secondary dentin ****
23. cuspal reduction (shoeing ) in complex amalgam restoration
a. 1-1.5 b. 2-3************ c. 0.5 -1 d.1.5-2
24.intracoronal bleaching. Which agent used
Sodium perborate
25. process of removal of one root of a tooth to save the other by leaving the crown intact
a. root resction
b. root amputation **
c. bicuspidization
26. tooth surface most prone to caries
a. pit nd fissure **
27. graft for grade II defect
28. long case about ectodermal dysplasia
29. common reason for failure of pit and fissure sealant
a. contamination during placement **
b. caries
30. radicular cyst always associated with
a. vital tooth
b. caries
c. non vital tooth**************
31. bone temperature during implamt placement
a. 42 c for 1 minute
b. 46 c for 1 mnt ***
32. amalgam used after root resection
a. high copper
b. zinc free ***
c. copper free
33. which tooth extraction aimed during serial extraction
a. primary canine ***
b. permanent canine
c. primary molar
34. cement used under composite
a. zinc phosphate
b. zinc polycarboxylate****************
c. light activated GI
35. case about calcifying odontogenic cyst. Cells seen
Ghost cells
36. Flare up technique used in
a. single visit **
b. large lesion
37.glass ionomer cement property
Fluride release
38. picture of an implant mask shown and asked what it is
39.active bristle brushing technique ( bass where bristles play an important roleand modified stillmas where
side parts of the bristles play important role)
40. long case about a denture patient with red coloured denture bearing area
a. epulis
b denture stomatitis ***
41. long case picture shown white patches in buccal mucosa 15 yr old child had exams last week.
Histopatholgy civatte bodies , hyperkeratosis etc
a. leukoplakia
b. lichen planus************
c. lichenoid rctn

Exam13
1)Maximum dose of Carbamzepine for trigemnal neurlagia disease :
A-100
B-500
C-1000
D-1200*******
2)Broad spectrum antibiotic for pedo pt are used when necessary.
what is the suseptiblity of microorganisms and potential of host toxicity
A-Wide range of microoganisms and high host toxicity ******************
B-Wide range of microoganisms and low host toxicity
C-Narrow range of microoganisms and high host toxicity
D-Narrow range of microoganisms and low host toxicity
3)Moderate to sever Hypermobile ridge what is the treatment:
A-surgical modification of rige *****
B-make denture with denture adhesive
C-make denture with balanced occlusion
D- use of scleroting agents
4)Pt comes to clinic complaining of loss of retention of her denture that she have done 1 year ago and wore it
daily without removing it what is ur treatment:
A-Vestibuloplasty
B-alveoloplasty
C-recovery treatment*******
5)Implant supported complete denture ,the pt complain of loss of retention , he said in the first 2 weeks of
denture delivery he had difficulty in placing it and then the retention started decreasing gradually, clinically
he have inflamed ridge , why the retention is decreased?
A-Loosness of ball abutment of implant
B-Non parallell placment of implant *****
(Sorry dont remmember the other options but those 2 were most convenice for the answer)
6)Pt came for prosthetic treatment , she has missing #24 #26#27#28 and #25 is left as peir abutment , it has
lingodistal inclination , MOD amalgam restoration and grade II mobility, what is ur treatment?
A-Extract #25 and RPD*****
B-FPD on 23-25 and RPD on 26 27 28
C-Overdenture RPD on 25
D-RPD
7)Pt cant open mouth , in morning she has 30 mints of stiffness , xray shows disc is internal (i dont
remmember the exact prescription of the disc) tmj is deviated when opening :
A- rhematoid arithritis*****
B-osteoprosis
8)Pt with paralysis of left side for 15 days including the eye, upper and lower lips , anterior third of the
tounge , cannot raise the eyebrow , which injury of facial nerve is this due to?
A-Injury to Parotid gland
B-After the origin or chorda tympani
C-Upper part of facial nerve****
9)thermomechanical compaction of gutta-percha is:
A-Mcspadden technique****
B-obtura
C-ultrafill
10)Sectional cast post used in which tooth:
A-insisors
B-canines
C-premolars
D-molars*******
11)Nitrous oxide is a relative contraindication in :
A-pt who can tolerate long appointment
B-Suppression of gag reflex
C-Pt emotionally distress*******
12)Attrition of 27 yr old and xray show pulp obliteration , his mother mentioned she has the same problem
and his brother also:
A-osteogensis imperfecta
B-dentinogensis imperfecta ****
C-amelogensis
D-odontoplasyia
13)Pt with attrition what is seen in xray :
A-Pulp obliteration
B-hypercementosis ******
C-external resorption
14)Newborn with red viscle on labial gingiva :
A-epestien pearl
B-bohns nodule****
15)Pregnant women with lesion that is very red and bleed on touching she had bad oral hygeine and
generlized bleeding on probing :
A-granuloma gravidarum***
(Don't remember the other options but this is the correct answer)
16)well formed round radioluceny in 2nd molar ramus region:
A-focal osteoprosis of bone marrow of jaw
B-Residual cyst***
17)In 6 yrs old child with red and swollen area behind 2nd primary molar what is the dx:
A-eruption cyst***
B-developmental cyst
C-abcess
18)Pt with swelling in floor of mouth bilaterrally including sublingual submandibular and submental area
A-Ludwigs angina***
B-cellulitis
19)Preferable impression charchtristics. :
A-long working time , short setting time***
B-long working time , long setting time
C-short working time , long setting time
D-short working time , short setting time
20)pt with incompetent lips , protruded mouth , and inscisors are in class 1 , what is the malocclusion pt has
:
A-Class 1 Premaxillary proclination ***
B-class 1
C-class 2 div 1
D-class 2 div 2
21)pt with missing upper 21 22 what kennedy class?
A-class 4 mod 1
B-class 3 mod 1****
C-class 2 mod 2
D-class 1 mod 1
22)What is the test used to measure rotation of the bur at a point on the axis of the tooth
A-Runout ****
B-concentricity

Exam14
1)Maximum dose of Carbamzepine for trigemnal neurlagia disease :
A-100
B-500
C-1000
D-1200*********************
2)Broad spectrum antibiotic for pedo pt are used when necessary.
what is the suseptiblity of microorganisms and potential of host toxicity
A-Wide range of microoganisms and high host toxicity *****************
B-Wide range of microoganisms and low host toxicity
C-Narrow range of microoganisms and high host toxicity
D-Narrow range of microoganisms and low host toxicity
3)Moderate to sever Hypermobile ridge what is the treatment:
A-surgical modification of rige ***********************
B-make denture with denture adhesive
C-make denture with balanced occlusion
D- use of scleroting agents
4)Pt comes to clinic complaining of loss of retention of her denture that she have done 1 year ago and wore it
daily without removing it what is ur treatment:
A-Vestibuloplasty
B-alveoloplasty
C-recovery treatment*************************
5)Implant supported complete denture ,the pt complain of loss of retention , he said in the first 2 weeks of
denture delivery he had difficulty in placing it and then the retention started decreasing gradually, clinically
he have inflamed ridge , why the retention is decreased?
A-Loosness of ball abutment of implant
B-Non parallell placment of implant*************************
(Sorry dont remmember the other options but those 2 were most convenice for the answer)
6)Pt came for prosthetic treatment , she has missing #24 #26#27#28 and #25 is left as peir abutment , it has
lingodistal inclination , MOD amalgam restoration and grade II mobility, what is ur treatment?
A-Extract #25 and RPD***************************
B-FPD on 23-25 and RPD on 26 27 28
C-Overdenture RPD on 25
D-RPD
7)Pt cant open mouth , in morning she has 30 mints of stiffness , xray shows disc is internal (i dont
remmember the exact prescription of the disc) tmj is deviated when opening :
A- rhematoid arithritis**********************
B-osteoprosis
8)Pt with paralysis of left side for 15 days including the eye, upper and lower lips , anterior third of the
tounge , cannot raise the eyebrow , which injury of facial nerve is this due to?
A-Injury to Parotid gland
B-After the origin or chorda tympani
C-Upper part of facial nerve*************************
9)thermomechanical compaction of gutta-percha is:
A-Mcspadden technique********************
B-obtura
C-ultrafill
10)Sectional cast post used in which tooth:
A-insisors
B-canines
C-premolars
D-molars*********************
11)Nitrous oxide is a relative contraindication in :
A-pt who can tolerate long appointment
B-Suppression of gag reflex
C-Pt emotionally distress*******************
12)Attrition of 27 yr old and xray show pulp obliteration , his mother mentioned she has the same problem
and his brother also:
A-osteogensis imperfecta
B-dentinogensis imperfecta ******************
C-amelogensis
D-odontoplasyia
13)Pt with attrition what is seen in xray :
A-Pulp obliteration
B-hypercementosis ********************
C-external resorption
14)Newborn with red viscle on labial gingiva :
A-epestien pearl
B-bohns nodule*********************
15)Pregnant women with lesion that is very red and bleed on touching she had bad oral hygeine and
generlized bleeding on probing :
A-granuloma gravidarum*************
(Don't remember the other options but this is the correct answer)
16)well formed round radioluceny in 2nd molar ramus region:
A-focal osteoprosis of bone marrow of jaw
B-Residual cyst****************
17)In 6 yrs old child with red and swollen area behind 2nd primary molar what is the dx:
A-eruption cyst**********************
B-developmental cyst
C-abcess
18)Pt with swelling in floor of mouth bilaterrally including sublingual submandibular and submental area
A-Ludwigs angina******************
B-cellulitis
19)Preferable impression charchtristics. :
A-long working time , short setting time*********************
B-long working time , long setting time
C-short working time , long setting time
D-short working time , short setting time
20)pt with incompetent lips , protruded mouth , and inscisors are in class 1 , what is the malocclusion pt has
:
A-Class 1 Premaxillary proclination **********************
B-class 1
C-class 2 div 1
D-class 2 div 2
21)pt with missing upper 21 22 what kennedy class?
A-class 4 mod 1
B-class 3***************
C-class 2 mod 2
D-class 1 mod 1
22)What is the test used to measure rotation of the bur at a point on the axis of the tooth
A-Runout ******************
B-concentricity
C-both
D-None of the above
23)Complete mandibular ridge impression that produces excess in the distal end of custom tray :
A-brown compound
B-alginate *****************
C-polyester
D-additional silicone
24)Posterior palatal seal area
A-in hard palate
B-in junction of hard and soft palate
C-In soft palate*******************
25)Which of the following commonly found in diabetes mellitus;
A-gum abcess
B-gingival recession******************
26)Dentin response to caries ;
A-sclerotic *****************
B-increase permeiablity
C-dissolution of peritubular dentin
27)How to disinfect an impression;
A-tissue surface of impression
B-tissue surface of impression and the bottom
C-tissue surface of impression , the bottom , and the tray******************
D-Bottom and the tray
28)Pt with pain the test reveals : percussion- no response, cold-no response , Hot- severe pain and ept
response at 80
control tooth : percussion-no response, cold-5 sec , hot-5 sec , ept- response at 25, what is the pulpal
diagnosis:
A-healthy pulp
B-irreversible pulpitis*******************
C-necrotic
D-reversible pulpitis
29)How to detect pit and fissure caries?
A-Visual inspection
B-Visual Inspection and probing by explorer******************
C-Visual Inspection , probing by explorer and xray
D-Visual inspection and xray
30)Pit and fissure caries detected by explorer by:
A-sticking of explorer in soft point
31)Which one if those is Growth centre :
A-synchondrosis ******************
B-epiphesial plate,
C-sutures
D-condyle growth
32)Pulpal cell that allow lineage of another cell ;
A-macrophage
B-ectomesenchymal cell *******************
C-lymphocyte
D-neutrophills
33)Takes aspirin 4 times daily which blood test ;
A-bleeding time ****************
B-coaogulation time
C-Pt
D-Ptt
34)What is the percentage of copper in Amalgam free of gama 2 :
A-2
B-(not sure 4 or 6)
C-10
D-13********************
35)Resorption in orthodontics by which cell
A-osteoclast*************
B-Osteoblast
C-Ameloblast
36)Hutchinson inscisors in which stage?
A-morphodiffrentiation*******************
B-Histodiffrentiation
C-Apposition
D-Proliferation
37)Pt with Prosthetic myteric valve and want to do surgical extarction what u do before the appointment
A-prophylactic antibiotic *****************
B-Nothing
38)pt with INR 2.8 and want to do surgical extraction
A-Proceed with vitamin k IV
B-Proceed only *********************
C-Stop for 3 days and replace by aspirin
D-Consult
39)H file better than k files by:
A)Positive rake angle****************
B)Negative rake angle
40)K reamer 10 cross section :
A-triangular****************
B-Rectangular
C-Square
D-Round
41)Not commonly used in endo dx:
A-ice
B-electric pulp test
C-hot water**************8
D-percussion
42)Diffrence between Gracey currete and universal
1-Gracey for specific tooth area , universal for all surfaces
2-Gracey has one side cutting , universal is both sides cutting
3-Gracey cutting end offset angle is 70 currete , universal is 85
4-gracey is semicircular cross section , universal is triangular
A-1 and 2************* as universal is 90
B-2 and 3
C-2,3, and 4
D-1,2 and 3
43)Pt with normal centric occlusion but have interferance in eccentric movement what is your treatment:
A-Grinding of lingually inclined maxillary teeth ******************
B-Grinding of lingually inclined mandibular teeth
Grinding of maxillary incisors insically
Grinding of mandibular inscisors insically
44)Rpd is best made of
A-cobalt chromium *********************
B-Gold
45)Anticariogenic sugar substitute is :
A-Xylitol*******************
B-Mannitol
46)What is the xray of choice to detect some of the unerupted teeth of 12 yrs old child:
A-Panoramic***************
B-Periapical
C-Bitewing
D-Occlusal
Exam14

1-The ideal gap distance for ceramic solder joint is:


a. 0.1 mm
d. 0.5 mm
b . 0.2mm*****************
C. 1,0mm
2-A patient made for himself a complete denture. After a few days he comes to you complain
ing from pain and white spots on the residual ridge and you do relief in that area and give him
ointment. After a few days he comes again complaining the same but in another area. The
main cause is:
a. Uneven pressure on the crest of alveolar ridge.****************
b. Rough tissue contacting surface of denture
c . Increase vertical dimension
d. Absence of balancing occlusion
3-GG1.Diameter used..
a.20
b.40
c.50******************
d.10
4-Calcification of primary tooth starts in :
1st trimester
14 weeks *****************
2nd trimester
3rd trimester
After Birth
5- after gingivectomy healing at
a. 1-7
b. 5-14******************
c. 14-21
6- 8years old pat. came to clinic has trauma before 3 days in x- ray no fracture of root and no priapical
pathosis what is the ttt:
a-partial poluptomy with formacresol ***************
b- DPC
C- polpectomy with caoh
D- complete polpoectomy with caoh or MTA
7- smoker pat.has missed lower 4 ants. and with slight class l mobility of 2 canines and he the dentist decided
to do RPD to what the type of lower major connector :
1 lingual plate****************
2 lingual bar
3 double bar
4 ???
8- case control
1 outcome **************
2 sample
3 risk
4 disease
9-pt.has "shamma
she used to bite on it for several years, recently she has a
some histologic feature related to
cancerous lesion:
a. SCC
b. verrocous carcinoma***************
10-Que about >>>>odontogensis imperfecta .
11-Que about >>>>> dentinogensis imperfecta.
12- the common cause of discoloration of single tooth :
a-endo ttt*******************
b-tetracycline
c-dentiogensis imp.
d-Amelogensis imp.
11-tips of file 20:
0.2 *******************
0.02
12-female pat came with previous endo ttt
With MD amalgam restoration and no enough walls what is the ttt:
a-Post core and crown *********************
b-I can't remember ???
13- case about diabetes pat. >>>>> gingival resseion
14-case about diabetes pat>>>>RPD
15-que. about PT Nnd PTT
16-female pat 24 years old with good healthy had orth treatmeant before 2 month and then she note that
when she brush her teeth there is bleeding from some teeth wht is the cause :
a- deep pocket
b-gingivitis ******************
C- hyperplastic gingival
d- enlarg in gingiva
17-10 years old child with congenital heart disease came for extraction of his lower 1st molar.
The antibiotic for choice for prevention of infective endocarditis is:
Amoxicillin 50 mg/kg orally 1 hour before procedure****
18-: Brushing technique in which sides of bristles takes active part
Modified stillman *********************
Charters
Bass
19- proxy brush >>>> embrasure type 2
20- the most Salivary gland disease (tumor) :
A. Pleomorphic adenoma.***
B. Adenocyctic carcinoma.
21- case about >>>Necrotizing sialometaplasia
22-Patient has swelling in submandibular area increase with
eating what is your diagnosis:
A. Ranula
B. sialothisis ***
C. mucocele
23- To plan the line-angles in the proximal cavity in a class II you use:
A. Straight chisel.
B. Bin-angled chisel.******************
C. Enamel hatchet.
d. Beveled chisel.
24-To plane the facial and the lingual wall of enamel, which enamel will use
A. Straight chisel.
B. Bin-angled chisel.
C. Enamel hatchet*****************.
25-hand instrument used to make retentive grooves or internal angles
A)Angle former ******************
B)Enamel hatchet
C)Chisel
26-success rate of implant :
>95*****************
85-95
<85
???
27-the bur that use to make groove in full veneer crown
28-least heat generated in :
a-carbide bur*******************
b-steel bur
c-diamond bur
d-titanium bur
29- Alter alginate time safely by what :
powder water ratio
water temperature******************
accelerator powder
30 -tow que. about poly vinyl silicon.
31- que. about gow gate tec. With pic .
32- pic about class 1 modification 2
33-which artery is branch from internal carotid :
1-Ophthalmic******************
2-Facial
3-Occipital
4-auriculotemporal
34-terminal branch of trigeminal nerve
Mental and insicive ******************
Long Buccle
Mental
36-fracture of angle of mandible u should take care about :
Facial n.
auriculotemporal*********************
37-opening an incision in a periapical abscess in a lower 1st molar, you open :
a-The most bottom of the abscess*****************
38- childe I think 7 years old has thump sucking what u will see I don't remember well :
Open bite and pot. Cross pite***************
39- que. about insicive cyst like a case
40- que .about a dye to detect the presence of cancer in the floor of the mouth and ask about what the other
matrial associated with it to make it more effective :
1.Methalyen blue
2-lugol********************
41-que. about middle meatus don't remember
42-but make groove in porcline Verne :
I chose Tapered
43-smooth of maxillary tuporcity >>> 12 blade
44-time of epthlizition********** 7-14 days
45-case 14 years old pat. stressed,
history of hepC i dont remember :
A-lichen planus *************
B-lichnoid reaction
C-aphtuos ulcer
D-lekoplakia
This case came with pic like lichnoid
46- the consecutive light of single tooth :
Bleaching ***************
Composite
Veneer
47-que about child Truman after 30 min. pic central with red pin point.
48-Rideal-Walker test is the test for detecting activity of:
a. Disinfection. ***
49-female pat. Look very old what the cause:
Extensive decrease occlusal vertical dimension
50-length of needle >>>0.4
51- about L.A in Childers >>>4.4
52-impaction of food in the proximal restoration :
Polymraiztion shrinkage
Improper wedging****************
53-about Maryland bridge which tooth replaced :
Maxillary central
Max. Lateral******************
54- the contraindication of functional appliance :
Maxillary protruded***************
Max. retruded
Mandibular protruded
Mand. Retruded
55- case about necrotizing sialometaplasia
56- case about AOT
57- case about denegrous cyst
58-difference between the alveolar epithelium and the gingiva epithelium is
a-Absence of stratum spinosum
b-Absence of stratum granulomatous
C-Absence of stratum cornium**********
59-the best diagnosis to know endo :
Hot
Cold *****************
Electrical
Precuon
60-case about irreversible pulpitsi and his treatment

??
62-Discoid cleoid instrument used in :
A) polishing amalgam before carving.
B) polishing amalgam after carving.
C) removing amalgam detaching.
D) used in carving amalgam.*****************

Exam15
1) Simplifi type canal enlargement by NiTi in or NiTi rotary file is
A// universal protper********************
B// reciprocal
C// revers s
2)Water irrigation device :
1. Dilute bacterial product**************
2. Remove plaque
3.Prevent plaque formation
3)During development what is the number of the pharyngeal arches:
a)5
B/ 6*****************
4)Stomodeum and foregut separated by:
1. Oropharyngeal (buccopharyngeal) membrane*********************
2. Ectodermal cleft.
5)enamel and dentin formation starts from
1. crown
2. cervical loop*****************
6) Pleomorphic adenoma its size is 1.5 x1.5 cm on posterior part of hard palat, what is ttt :
a.Enucleation only
b.Radiotherapy
c.Chemotherapy and enucleation
d.Resection of periostuim.**************
7) Concentration of CHX in mouth washes ? 0.12%
8) nowadays best management complete edentulous for mandible
a. complete removable denture
b. over denture with 2 implants **********************
c. over denture with 4 implants
9) Min LA for child 2% lidocaine with 100000 epinephrine
a.9mg/day
B.4mg/day *********************
c.5mg/day
d. 7mg/day
10) What type of L.A have the slowest
onset .
-1procaine**************
-2lidocaine
-3bupivacaine
-4mepivacaine
11) function of vasoconstriction in L.A?
1-increase intensity of la**************
2- decrease intensity
12)Long case about a dentist doing a perio surgery and the patient shook and the instrument broke.How to
retrieve it:
- Schwartz periotriever************8
13) Cavity etching before applying GIC is ?
A ) Phosphoric acid 10 seconds.
B ) Polyacrylic acid 60 seconds.
C ) Polyacrylic acid 10 seconds.******************
D ) Phosphoric acid 60 seconds.
14) Lower impacted canine use
-Occlusal xray****************
-Panoramic
15) question about deep pit and fissure in a 25 year pt has mild to moderate risk caries what tt?
A -no tt
b- Preventive resin restoration
c- class 1 restoration
d sealant****************
16) when to do initial survey
a in diagnostic cast ****************
b in working cast
c after final impression
d before insertion partial denture
17) On a central incisor receiving a full ceramic restoration during finishing of the shoulder finish line
subgungivaly
a Radial fissure
b Straight fissure
c Round
d Diamond end cutting************
18) Diffrence between Gracey currete and universal :
1-Gracey for specific tooth area , universal for all surfaces
2-Gracey has one side cutting , universal is both sides cutting
3-Gracey cutting end offset angle is 70 currete , universal is 85
4-gracey is semicircular cross section , universal is triangular
A-1 and 2*****
B-2 and 3
C-2,3, and 4
D-1,2 and 3
19) patient with complete denture has painful lesion in floor of mouth diagnosed as carcinoma what should
you ask the patient
- is the denture ill fitting
- history of alcohol and smoking************
-
-
20) need for physiologic occlusion
A esthetic and support
B function
C occlusal stability**********************
21) best maxillary arch shape for stability and retention of denture
a u shape******************
b tapering
c flat
d high vault
22)bacteria responsible for intiation and progression of caries
- strep mutans and lactobacilli******************
23) xray of odontoma
24) pic of man with complete denture not happy with esthetics after teeth arrangement everything else was
good
- mid line shift of teeth*****************
- wrong canine line
25) best xray for proximal caries
- bitewing
26) max dose of carbamezapine
-100
-500
-1000
-1200**************
27) Pt got intrusion after trauma, x ray shows progressive root resorption, diagnosis:
A- Replacement resorption
B- Surface Resorption
C- Cervical resorption
D- Inflammatory resorption****************
28) temporary cement used in temporary crowns to :
1- provide retention for crown .******************
2- distribute force with long axis of tooth .
3- act as seal against margin
29) why gic used as filling in carious tooth undercontrol
- fluoride release*************
30) tooth with sinus tract diagnised as
- acute exacerbation of chronic abscess******************
- acute apical periodontitis
31) goal of preventive dentistry program
32) mandibular foramen in children is:
a. below the level of occlusal plane***************
b. Above the level of occlusal plane.
c. Anterior the level of occlusal plane.
d. lateral
33) Vestibuloplasty which tech. pt. having maxillary complete denture shallow vestibule good labiolingual
vestibule
a. Clark- tech.
b. (Kazangachi- tech.)
c. Transposition- tech.
d. Obweg tech.*****************
34) Patient ,with amalgam containing zinc restoration for a simple class one cavity ,arriving your clinical
with pain after one month:
1.occlusal trauma
2. Pulpal involvement.
3.delayed expansion.***********************
35) Rideal-Walker test is the test for detecting activity of:
1. Disinfection*******************
2. Antibiotics.
3. Sterealization by dry heat.
4. Sterealization by wet heat.
36) In cavity what about of use dry sterilize disinfected solution
A.useful in deep cavity
B.cause pulp irritation
C.general not useful****************
37) best disinfection method
- immersion in solution
-ultrasonic bath***************
38) 56 yrs. old patient has a sharp pain in right side of face, this pain is painful and aggressive,
when touching left cheek patient feel pain
a. bell palsy
b. trigeminal neurologia********************
c. psychological pain
d. . syndrome
39) how alveolar height increases
-eruption of teeth********************
- movement of skull base
40) Radiographic diagnosis of a well defined unilocular radiolucent area between vital
mandibular bicuspids more likely to be:
1. Residual cyst.
2. Mental foramen.************************8
3. Radicular cyst.
4. Osteoporosis
41) Question about anatomical structure in the mandible can appear as pathological problem and let the
dentist confused :
A/Mental foramen *********************
B/Retro molar pad
42)Which of the following is most
important in determining the
prognosis for a tooth?
A. Probing pocket depth
B. Bleeding on probing
C. Clinical attachment level***************
D. Level of alveolar bone
43) inferior alveolar nerve terminates as
- mental
- incisive
-mental and incisive******************
44) The main purpose of flux in soldering is to
A. dissolve surface oxides and prevent further oxidation.***
B. prevent recrystallization and grain growth.
C. prevent oxidation and lower the melting range of the solder.
D. dissolve surface oxides and lower the melting range.
45) Most difficult position in impacted lower 8
Mesio angular
Disto angular***************
Vertical
Horizontal
46) The most ductile and malleable metal is:
a) Sliver.
b) Gold******************
c) Copper.
d) Platinum.
47) if the degree of fusion of porcelain is1150c and alloy melt 920 c what is the degree of fusing them
together
to make porcelain fused to metal crown
A.730
B.920 ************
C. 1150
48) CSF composition
1-high in protein
2-high in glucose********************
3-glycoside transferase
4-enzyme
49) What is the maximum of 2%lidocaine with 1:100000epinephrine 80 kg ptt.?
50) Maryland bridge is an example of
A. Conventional bridge
B. Telescopic bridge
C. Resin bonded bridge*****************
D. Cantilever bridge
51) Pt with periodontal problem & u want to do ortho , what type of force
A) high
B ) interrpted
C) light *******************
D) tipping
52)For grade 2 furcation involvement what type if bone graft:
-corticocancellous
-Cancellous
-Cortical
-Freeze dried bone****************
53) Contraindication of gingivectomy
A-suprabony pocket
B-suprabony periodontal abscess
C-gingival enlargement
D-defet in alveolar bone****************
54) Post graduated student use MTA, the prognosis depends on prevents:
(RetrogradeMTA filling)
1. Immediate suture.
2. Using a flab.
3. Disturbance during closure of wound****************
55) Best media for the avulsed tooth:
- milk*********************
- saliva
- saline
56) 7 yr old Trauma intrusion of 2 central permanent incisors with open apices:
a . apexogensis
b . surgical extrusion
c. orthodontic extrusion.
d . observation********************
57) What is the goal of maintenance therapy
- To prevent recurrance of disease*******************
- Evaluate tissue response
-
-
58) What is the Difference b/w standardized GP and Conventional GP ?
- standardized GP has same size as file
59) Grade II furcation treatment
- GTR ***************
- Hard tissue
-
-
60) Lower RPD with 6mm space available lingually, which major connecter
- lingual bar
- lingual plate********************
- double lingual bar
-
61) Best hand washing technique
- scrub your hands
- use alcohol when soiling obvious
- use soap bar****************
- do not dry your hands
62) Age at which second molar erupts
- 12*****************
- 14
- 10
63) Most common teeth with dens invaginatus
- upper central
- upper lateral**************
- upper premolar
-
64) Position of practitioner's arm while giving anaesthesia
- away from patient*************
- resting on pt chest
-
-
65) You give PSA block but mesial root of 6 is not anaesthesized. what should you do?
- repeat PSA
- anaesthesize ASA
- give buccal infiltration***************
-
66) 10 year old patient comes to clinic, child cannnot name colors, suffers from seperation and can't say his
name.This stage similar to which age?
- 3 yr*****************
- 4 yr
- 10 yr
-
67)What is the easiest tooth for RCT?
- overfilled
- underfilled
- tooth with weeping canal***************
- tooth with silver points
68) Which canal is usually missed during RCT of lower anterior
- buccal
- lingual*******************8
69) Split dam techniques used for
- single tooth with crown
- posterior teeth for Rct treatment
- bleaching
- teeth with bridge*******************
70) MAterial for bleaching of non vital tooth
- hydrogen peroxide
-
-
71) Reason for discoloration of crown of non vital tooth
- blood by-products****************
- pt consumes too much coffee
-
-
72) what is the best method to detect pits and fissure caries?
- visual inspection*************its with blunt probe
- use of sharp explorer and visual inspection
- visual inspection and xray
- use of sharp explorer, visual inspection and xray
73) 14 months baby with bluish fluctuant swelling treatment
-
74) A case about an old woman with thickened mandible and multiple radioopacities... diagnosis.
- fibrous dysplasia
- paget's disease*******************
- 2 more options
75) what is the new treatment approach for edentulous mandible
- complete denture
- overdenture with 2 implants**************
- overdenture with 4 implants
76)The test for testing the rotary tip passing the same point
- Concentricity
- Runout and concentricity
- runout**********************88
- None of above
77) Resorption in ortho depends on
- magnitude*****************
- duration

Exam17

My questions
1. LA cotradicted in heamophilics
Inferior alveo;ar nerve block
2. Comp of gutta purchase
70 zno 20 gp
3. Sugar that inhibits caries
Xylitol
4. Slowest onset LA
Prilocaine
Lidocaine
Bupivicain*************
5. Heat test 15 sec cold 20 sec
Irrev pulpits
6. Flap and cheek retractor
Minnesota
7. Property of cement under amalgam
High mod of elasticity
8. 15 85 10 35
Blade angle?35
9. Overlapping of image
Improper horizontal angulation
10. Dye to detect caries
propyl glycol
11 .pic of IAN technique
12. Pic of fibroma
13.pic of amelogenesis imperfect
14. Shiny hair less no. Of teeth decreased sweating
Ectoermal dysplasia
15.disinfectant for alginate
7%glutaraldehyde
16. Economical impression material
Alginate
17. Tuberosity bloc
PSA
18. LA given to pregnant woman
Lidnocaine with vasoconstrictor
19. Amt of fluid for 4yr old
.5
20. Angle of blade while scaling
70 80
21.fluride varnish
5 naf
22.schick test
Diptheria
23.autoclave
Steam under pressure
24.most common blade
15
25.antibiotic inhibiting cell wall
Penicillin
26.palate composed of
Palatine and maxillary
27.healling after scaling and root planing
Long junction all
28.least accurate test
Electric
29.inferior alveolar branch of
Posterior branch of manned
30. Pt with complete denture pronounced for as v
Maxillary anterior teeth had placed too far from superiority and anteriorly
31.time between the first onset of HIV and appearance of acute symptoms
9-11
32 Periodontal ligaments in middle third of root
Oblique
33.Child with previous history of minor Trauma with excessive bleeding we do test the result is prolonged
PT and slightly increased clothing time diagnosis is
Vit j deficiency
34.child presents with bilateral swelling of submandibular area, what could be the disease
Cherubism
35.non -absorbable suture
Silk
36.chouce of local anaesthesia technique influenced by
Bone structure
37.acyclovior dose
200mg 5 time a day
38.limitation use of apex locator
Narrow canal
Short canal
Curved canal
Open apex**
38.aggressive brushing causes
Abrasion
39.inst for cutting bone
Ronger
40.tooth surface suceptable to caries
Occlusal of lower 1st molar
41.q on Kennedys
42.location of finish line in abt tooth determined by
Clinical crown lenth**
43.types of steel in dental inst
Satinless steel and carbon steel
44 Pt unable to say ah
Vagas nerve
45. Critical pH of saliva in which saliva demineralises
5.5
46.increased rate and depth of respiration
Hyperventilating
47.tt of pericorinitis
Rinse and clean
48.contraindication of procelin
Bruxism
49.gingivectomy
Removal of pseudo pocket
50.angle of root amputation
Acute angle
Exam18

1 - restoration contraindicated with mouth breather : GIC


2 -best type of bone for implant : type 2
3 - least type of bone for implant :type 1 or 2 or 3 or 4
4 - case about Pt mised upper right central & asked about the abutment : 12 & 21
5 - base under composite : ca oh or zn ph
6 -picture for dentigerous cyst
7 - picture for fibroma
8 -picture dentinogenisis imperfacta
9 -radicular cyst associated with : vital or non vital tooth
10 -type of regenerative non graft technique : GTR
11 - pacteria found in new born mouth : S . Salivaris
S. mutans
12 -when we do RCT the 1st test we do : cold
hot
percusion
electric
13 - check test for : diphitaria
14 - how to detrmine the osseous defect of bone
15 - activation of perio instrument blade should be at angle with facial surface of tooth : 45 -90
16 - number represented the angle of the blade
17 - number represented angle of cutting edge
18 -component of elevator : handle shank and blade
19 - definition of pulpotomy : remove of cronal pulp
20 - finishing bur for composite have : more than 12
21 -age must parents take children for ortho ttt : 5 - 6
or
**7-8
22 - mobility of the tooth without loss of bone suuport : primary occlusal truma
23 - to increase setting time of alginate :hot water
cold water
temprature
24 -excssive use of dental flose : cause reccision
affect periodontal health
cause laceration **************
25 - supra persteal injection indicted in : when inject in infected site or in hard bone
26 -drug cause gengival enlargment : phentoin
27 - drug decreas saliva : anti cholenergic
28 -over hanging of amalgam afect : periodontal health
29 - shape of rest seat is concave or triangular
30 -healing after gengivitis occur after : 7-14 days
31 -dis infection of GP : sodiom hypo clorid
32 - hemophilia B : factor 9
33- acces cavity for upper lateral : trianglar
ovoid ***********
34 - pemphigus ChCh by : akanthosis
acantholisis***********
35 - externa resorption :necrotic
36 - conditioning for GIC :10 % poly acrylic
37 - nerve supply of TMJ ..auriculotemporal nerve
38 - arterial supply of TMJ ..superficial temporal artery and deep auricular artery branches of
maxillary artery
39 - best metal used in implant : titanium
40 - function of post palatal seal : aid in retention of denture
make relining of denture easy
41 - spread airborne pathogens : Aerosols
42 - which event occur during dentinogensis : matrix & proteo glycans maturate with colagen fibers
43 - question about cases we should use prophilactic antibiotic

Exam19
2. Polyether impression materials:
a. Are less stable dimensionally than polysulfide rubber.
b. Are less stiff than polysulfide rubber.
c. Can absorb water and swell if stored in water. ***
3.When does child should be first exposed for using tooth brush:
a. As eruption of first tooth. ***
b. One year old.
c. Two years old.
d. Primary school year.
4.Optima water fluoridation:
a) 0.5 0.8 mg\liter. ***
b) 0.2-0.5 mg\liter.
C) 2-3 mg\liter
5.The incisal reduction for a metal ceramic restoration should be:
a. 1.5 mm.
b. 2 mm. ***
c. 3 mm.
d. 4 mm.
6.It has been proven that amalgam restoration has the following characteristics:
a. Micro leakage decrease with aging of the amalgam restoration.
b. It is the least techniques sensitive of all current direct restorations.
c. High dimensional changes.
d. a, b and c.
e. a and c.
f. a and b. ***
g. b only.
7.When polishing the amalgam restoration:
a. Avoid heat generation by using wet polishing paste.
b. Wait 24 hours.
c. a and b. ***
d. b only.
e. a only.
8.Plaque consists of:
a. Bacteria
b. Inorganic material
c. Food
9.Teeth with RCT and you want to use post, which post is the least cause to tooth fracture :
1. Ready made post.
2. Casted post.
3. Fiber post. ***
4. Prefabricated post.
10.The best restoration for max. central incisor that has received RCT through conservatively prepared
access opening would be:
a. Post-retained metal-ceramic crown.
b. Post-retained porcelain jacket crown.
c. Composite resin. ***
d. None of the above.
11.After trauma a tooth becomes yellowish in color, this is due to:
a. Necrotic pulp.
b. Irreversible pulpitis.
c. Pulp is partially or completely obliterated.****
d. Hemorrhage in the pulp.
12.Hairy trichoglossia may be caused by:
a. Broad spectrum antibiotic.
b. H2O2 mouth wash.
c. Systemic steroid.
d. Heavy smokers.
e. All of the above. ***
13.. After extraction a molar you found a hard tissue at the furcation like pearl . what is it:
OR
Tooth # 36 planned to extraction on x-ray no PDL after extraction u found lesion like pearl on furcation
whats the lesion:
a. Enamel pearl***
b. Enostosis
c. Hypercementosis
14.After finish class V glass ionomer cement we do finishing with:
1. Pumice slurry.
2. Aluminum-oxide disc.*****
3. Prophy paste with fluoride
15.( this question was modified with few more additional signs seen radiographically)
Radiopacity attached to root of mandibular molar:
a. Ossifying fibroma.
b. Hypercementosis. ***
c. Periapical cemental dysplasia.
16.Acute periapical cyst and acute periodontal cyst are differentiated by:
*We distinguish between periapical and periodontal abscess:
a. Vitality test. *** (Vitality of the pulp)
b. Radiograph. (X-Ray examination)
c. Clinical examination.
17.( the answers were bit modified but the question was same) Treatment of internal resorption involves:
a. Complete extirpation of the pulp to arrest the resorption process. ***
b. Enlarging the canal apical to the resorbed area for better access.
c. Utilizing a silver cone and sealer to fill the irregularities in the resorbed area.
d. Filling the canal and defect with amalgam.
e. Sealing sodium hypochlorite in the canal to remove the inflammatory tissue necrotic in the area of the
resorption.
18.Solitary bone cyst management:
a. Anti-inflammatory and follow up.
b. Curettage and close. ***
c. Marsupialization and antibiotic
d. No active management.
19 .When polishing amalgam restoration:
a. Avoid heat generation by using wet polishing paste.
b. Wait for 24 hours.
c. a & b. ***
d. a only
e. b only
20.Endomethasone is a root canal sealer that:
a. Dissolves in fluid so it weakens the root canal filling.
b. Very toxic containing formaldehyde.
c. Contains corticosteroids.
d. All the above. ***
21.Tooth germs of primary teeth arise from:
a. Dental lamina. ***
b. Dental follicle.
c. Enamel organ.
d. Epithelial cell of malassez.
22.Dentinogenesis imperfecta have all EXCEPT:
a. Broken enamel.
b. Blue sclera.
c. Broken bone.
d. Supernumerary teeth. ***
23.Firm, fixed neck nodes are most to be detected in association with:
a. An ameloblastoma.
b. A basal cell carcinoma.
c. An odontogenic fibroma.
d. A squamous cell carcinoma. ***
24.Osteogensis during endodontic surgery aimed to prevent:
A. fibrous in growth. ***
B. growth factor.
C. formation of blood.
25.( the question was bit modified adding a term cement trail in radiograph)
After u did RCT to your pt. he came back to the clinic after few days with sever pain on biting, you did x-ray
and it revealed that the RCT filling is very good, but u saw radiopaque, thin ( film like ) spot on the lateral
border of the root what is the most probable diagnosis?
A) Accessory canal.
B) Vertical root canal fracture. ***
C) Perforation
26.Patient came complaining of severe pain on biting, related to a certain tooth. Upon examination no pulpal
or periodontal findings, and pulpal vitality is positive, your Dx:
1) cracked tooth syndrome.***
27.Patient comes to you with edematous gingiva, inflamed, loss of gingival contour and recession, what's the
best tooth brushing technique?
A. Modified bass.
b. Modified stillman. ***
c. Charter.
d. Scrub.
28.Stock trays compared to custom trays for a removable partial denture impression:
A. Custom trays less effective than stock trays.
B. Custom trays can record an alginate impression as well as elastomeric impression.
C. Custom trays provide even thickness of impression material. **
29.65 years old black man wants to have very white teeth in his new denture what should the dentist do:
a- Put the white teeth
b- Show the patient the suitable color first then show him the white one.****
c- Convince him by showing him other patients photos.
d- Tell him firmly that his teeth color are good.
3o.Mucocele The best ttt. is:
a- Radiation
b- Excision. ***
c- Chemotherapy
d- Cauterization
e- leave it
31.Sharpening the curette and sickle, the cutting edge should be at angle:
a- 50-60
b- 70-80. ***
c- 80-90
d- 60-70
32.Follow up of RCT after 3 years, RCT failed best treatment is to:
a) Extraction of the tooth.
b) Redo the RCT . *** ( Redo = Retreatment ).
c) Apicectomy.
33.At which temperature that gutta percha reaches the alpha temp.:
a- 42 - 48 c ***
b- 50 - 60
c- 70 -
34.Schick test is an intradermal test for determination of susceptibility to:
a. Diphtheria hypersensitivity. ***
b. Tuberculosis hypersensitivity.
c. ??? lepsron.

Exam20

1. During perio case curette gets broken instrumemnt used to retrieve it :


shwartz perioretriver.
2. Upper dentulous lower only anteriors which also have pdl compromised nd mobile ,Supra erupted.. ttt??
swingLock*** ,over denture,lingual plat.
3. PT on anitcoagulant therapy ... Surgeon must confirm that
PTT INR 1-1.5
PT Inr 1 -1.5
PTT inr 2-2.5
PT INR 2 -2.5****************
4.' A 21yr old boy c/o' SHIFTING TEETH' present with enlargmnt of mandible..also give history dat his
fathr had a leison and surgically removed frm jaw..wit R/g showing radiolucent leisons on both side of
mandible and on anteriors too..
Keratocystic odontogenic tumor*
Ameloblastoma*
Dentigerous cyst.
Radicular cyst
5. Pdl weak teeth type of orthodontic force used to move :
light***,heavy,tipping ..
6. For a screening test jr dr took impression best possible way to disinfect:
1%Naocl*, H2O2, 7%glutaraldehyde*,NaCl
7.ceramic primer is a:
Bonding agent* ,resin,..
8. Type of cement used for bonding procelain veneers..
Resin ionomer*, Znpoly, Znphosphate..
9. Main advantage of gic on caries ..
Flouride releasing **
10. advantage of Znpoly over Zn phosphate
Biocompatability**,setting tim,...
11. function of rest on rpd
12. quality which act against adhesiv food nd rpd:
retension**, stablity,...
13. Pic of compound odotoma
15. questn on cohort study...
16. Powerd tooth brush invented :1939**
17. wen we check occlusion for amalgam.. aftr carving, jst before dismissing da pt,aftr 1 hr,aftr 1 day.
18. pic showing anterior opn bite and asked abt its etiology :
thumb sucking**,anterior opn bite, bruxism...
19. 7yr old wit pacifier habit possible features:
anterior opn bite wit posterior crossbite*
20. longest tim to present to a dentist wit an avulsed tooth to sav it...
1hr,********** 1 day, 12-20hr...
21. best availabl medium for avulse tooth:
Low fat milk**,saliva,water
22. Why topical fluoride application doesnt cause fluorosis:
teeth already calcified**
23. pt hav allergy to ester type la so decide to giv prilocain color of it :
yellow
24. pt wit trismus ,technique used to give La:
vazirani akinosi
25. biocompatible material for implant:
titanium( bt da actual question is som twisting best bio chemical featureof implant)
26. mentaly retarded child wit multiples caries nd al:
restore al togthr under GA**
27. endo instrument standardisation is based on:
width of da tip** ,height,lenght..
28. Qstn on incisive canal cyst
29. Progress of perio surgery determinened by formatn of:
jn epithelium
30. established gigivitis aftr: 14-21 days
31.caries is initiation and progress by strep mutans nd lactobacillus.
32. main use of floss.:
destroy interdental plaque matrix.
33. Side of bristles activated on:
modified stillman***********charter,bass
34 .qstn on ectodermal dysplasia
35. hematoma aftr psa block:
pterygoid plexus of veins
36.managmnt of pseudo class3,,,,,,,removal of cause
37. questn on sequalae of pulpitis
38. feature see in apical periodontitis: a.inflammation with pmn infiltration **
b.Superimposed wit periodontal infections .
39.Bizygomatic width measured by
a.willis gauge b.facebow************
40. in flat ridge case teeth selected:
Zero degree or cuspless teeth**********
41. pic on congenitaly missing lateral incisor.
42. Wat happnd if da holes in rubbrdam are very closer :
inter dental papillae protrude frm beneath da dam.
43. For rubbrdam clamp to be ideal placed :
All four points of jaws shud contact da tooth gingival to contour *
44. Performed aesthetic crown material used: polycarbonate **,gic ,composite.
45. Branch of ICA: Opthalmic**, facial..
46. resorption by which cell: fibroblasts, osteoclast**, osteoblast .
47. Management of gingival enlargment: gingivectomy
48. Drug dat might cause gingival hyperplasia
Phenitoyin*,nifidipin, verapamil.
49. Peg lateral defect occurs in which stage:
morphodifferentation**, initiation,apposition
50. Old pt delivered wit CD came back complaining it get dislodged while moving tongue ,no other problms:
over extended lingual flange**, cramped tongue..
51. question on system B :
(I choose da option) wave condensation *
Obtura1, obtura 3..
52. Child given tetracyclin antibiotic frm birth nd during 1st yr of age..teeth dat might be affected... centrals
an first molars
52. Antibiotic used in periodontitis...tetracycline***, cefazoline ,sulfanamides..
53. 12 yr old pt with high caries susceptabilty ...mentioned a few teeth with non cavitated nd incipient
lesions.. no catching or R/g evidence...mangement except...
a. P.R.R
b. fluoride application
C. Gic
D. amalgam************
54. qustn abt properties of clasp
55. Property of NiTi:
shape memory and elasticity**
56. Tim for chlorhexide mouth rinse:
60 seconds, 30 seconds************, 2minute.
57. If 2 canals in mandibular centals chances of finding it on:
Lingual* ************, buccal, mesial, distal
58. A case of Recent radiation therapy presents wit infection followed by resction of jaw.. probable cause:
osteoradionecrosis
59. difference between normal and standardised gp used in endo.standardized gp is same the size of the
file
60. Common feature find on periodontuim of diabetic pt:
gum abscess*,
61. On opg apex of teeth not visible.. so on repeat wat measures taken to prevent dz error.
Head tilt upward
Head tilt downward
Lip placed on bite block
62. best irrigant in rct is sodium hypochlorid

Exam21
1. Proxy brush with which type of furcation:
a. Furcation Grade 1
c. Furcation Grade 3************
b. Furcation Grade 2
d. Furcation Grade 4
2.the way to remove mucocel is....exisision
3.what kind of periodontal probe is used in the furcation area : nabers probe
4-silicosis disease occures in: glass factory workers
5-composite restoration can be done after completion of bleaching by : 1 week ..
6-best matrix for SMALL class 2 restoration:
a tofilimre ***********b. celluloid c.. copper bond
6--21 y old pt who has iron deficiency anemia + difficulty swallowing with examination of barium sulphate
you found :
a..plumer Vinson*********** s b..iron defeciency anemia c.. burning mouth s
7--Autoclave relative to 100 degree temperature in the dry oven
a..less time********** b..same c.. slightly higher
8---management of tuberosity fracture during extraction of max molar is:
a..replace &suture************
b..remove &suture c..replace and suture by wire
9---during max 3rd molar extraction the tuerosity fracture ..it was farmly attached to the tooth ..what 2 do?
a...remove t with the tooth
b..splint the tooth 2 the second molar then reextrat after 6 weeks******************
10--root most cmmonly pushed in max sinus ? upper 1st molar palatal
11--the number of pharyngeal arches??
a..5 b...6 ***********
12--in order to decrease the gastric secretion:
a..histamine a b..histamine b c..anticholenergic***************
13---polyether impression:
a..less stable dimensionally than poly sulfide
b..can absorb water and swell if stored in wate************r
14--custom tray is preferred than stock tray :
a..thickness of impression of material will evenly spread by custom tray
15--ttt of internal resorption involves :
a..enlarging the canal apical to the resorbed area for better access
b..silver cone and sealer
c..filling the canal with amalgam
d..complete exirpation of the pulp to arrest the process*********************
e..sealing sodium hypochlorite in the canal
16---pt sharp pain during toutching of the cementum.by probing pain is present for short duration ?
a..reversible pulpitis b..irreversible
c..dentinal hypersensitivity*************
d..periapical abscess
17--A patient had pulp capping two months back, returned with pain, but no radiographic changes
are observed after 2 months. Reason could be:
a. Apical periodontitis b. Secondary apical periodontitis
c. Pulpitis*********** d. Periapical abcess
18--A 20 years old patient has avulsed tooth for 60 minimum. The management to return vascularity
of the tooth is:
a. Scrap the surface of the root
b. Place the tooth in sodium sulfide of 1 0%
c. Place it in sodium floride then sodium sulfide.**************
19--Most frequent cause of fainting in dental office:
a. Vaso-vagal shock************** b. Diabetes
c. Fear
19--An 8 year old child, suffered a trauma at the TMJ region as infant. Complaining now from
limitation in movement of the mandible. Diagnosis is:
a. Sub luxation b. Ankylosis***************
c. Trismus
20--A patient made for himself a complete denture. After a few days he comes to you complain
ing from pain and white spots on the residual ridge and you do relief in that area and give him
ointment. After a few days he comes again complaining the same but in another area. The
main cause is:
a. Uneven pressure on the crest of alveolar ridge.*************
b. Rough tissue contacting surface of denture
c . Increase vertical dimension
d. Absence of balancing occlusion
21--Formocresol when used should be:
a. Full saturated b. Half saturated
c. Fifth saturated d**********. None of the above
saturated concentration
22--The posterior seal in the upper complete denture serves the following functions:
a. It reduces patient discomfort when contact occurs between the dorsum of the tongue and the
posterior end of the denture base
b. Retention of the maxillary denture
c. It compensates tor dimensional changes which occur in the acrylic denture base during processing
d. A type of occlusion which is similar to the occlusion of the natural teeth
e. a and b************
23--What is the treatment for a patient with systemic candidasis, who is on a long term antibiotic?
a. Amphotrecin B
c. Nystatin
b. Fluconazol*************
d. None of the above
24--Indirect composite inlay overcomes the direct composite by:
1 . lnsusffition polymerization 2. Good contact proximaly
3. Gingival seal 4. Good retention
a. 1 -2-3***********
c. 4-3
25--AH26 composed of:
CAOH B..Epoxy resin************ c..ZOE
26--Parotid duct opens ..upper second molar********
27--Amalgam restoration with post and core in the posterior teeth is decided by:
a. Canal curvature b. Canal length and diameter
c. Amount of crown destruction************ d. All the above
28--Distal surface for first upper premolar ,contact with the neighbor teeth :
a. In the middle with buccal vastness wider than lingual
b. In the middle with lingual vastness wider than buccaly***************
c. Mesial contact in the middle third
d. Contact more cervical but still in middle third
e. The crown is little narrower in lingual than buccal
29--Pacifier habit will cause:
a. Crowding of tooth
b. Classll l malocclusion
c. Anterior open bite with posterior cross bite***************
d. Skeletal malocclusion
30--A child 3 years old came to clinic after falling on his chin. You found that the primary incisor
intruded the follicle for the permanent incisor. What you will do?
a. Surgical removal of the follicle b. Leave it
c . Surgically removal of the primary incisor************** d. None of the above
31--FPD bridge returns to the dentist from the lab with different degree of color, although the
shade is the same. The probable cause is:
a. Thin metal framework
c . Thick opaquer
b. Different thickness of porcelain****************
d. Inadequate firing of porcelain
32--Which of the following is used to detect the inter proximal caries of anterior teeth?
a. Periapical X-ray film************* b. Bitewing X-ray film
c. Occlusal X-ray film d. None of the above
33--Calculus induce further periodontal lesion due to:
a. Directly stimulates inflammation
b. More plaque adhere to it
c. Calcium deposition will cause gingival irritation
d. All the above*****************
34--A patient Is on 10 mg corticosteroids (prednisolone) for months, needs dental extraction. You
will:
a. Give antibiotics.
b. Double doze the day of extraction.***************
c. Double doze one day before, the same day, and day after surgery.
d. Take no action
35--To provide maximum strength of amalgam restoration the cava-surface angles should place
all EXCEPT:
a. Approach 75 with outer surface****************
c. Be supported by sound dentine
b. Approach 90 with outer surface
d. Be located in area free of occlusal stress
36--A patient has maxillary posterior partial denture with porcelain teeth He then lost the mandibular
posterior teeth. Now what type of teeth are used for mandibular partial denture:
a. Porcelain b. Acrylic
c. Metal ******* or acrylic reinforcedd. All the abov
37--A child patient with ob/itration in the central permenant incisor. What will you do?
a. RCT b. Pulpotomy
c. Pulpectomy d. Careful monitoring***********
38--How can you test crack tooth?
a. X-ray b. Electric test
c. Ethylene dye test*************. d. Percussion test

exam22
1) Geographic tongue is seen in Pt. with:
a. Diabetes. ****************
b. Iron deficiency anemia.
c. Pemphigus.
D. none
2)histology of nicotina stomatitis : hyperkeratosis and acanthosis
3) Pt came to u with sublingual space infection, change in colour of mucosa of floor of the mouth. The
tongue is slightly elevated. How will u do the incision for drainage ?
A. Extra orally parallel to lower border of the mandible.
B. Intraorally parallel to Whartons duct *******************
D. Intra orally between mylohyoid muscle
4) An anterior fixed partial denture is contraindicated when:
a. Abutment teeth are not carious.
b. An abutment tooth is inclined 15 degrees but otherwise sound.
c. There is considerable resorption of the residual ridges. **************
d. Crown of the abutment teeth are extremely long owing to gingival recession
5)main use of dental floss
1.remove calculus
2.remove overhang
3.remove interproximal plaque**
4.remove food debris
6)why calculus should be removed in perio diseases.
A. To avoid plaque adherence ***************
B. Because its an etiological factor of periodontal disease
C.it irritates tissues
D.to maintain good oral hygiene
7)Phenytoin not used with.... metronidazole true
8)Galvanic shock
A put seperating media
B put varnish
C change restoration
D wait**** treatment depending upon symptoms wait varnish than change restoration
9) Pt. has RPD with distal extension opposing to natural, he now with lost lower posterior teeth if he needs
RPD and upper teeth with porcelain type of teeth in lower:
1. Porcelain
2. Acrylic
3. Porcelain with gold occlusally
4. Acrylic reinforced**************
10. When doing a study. Participant should get:
a. Written informed consent in native language****************
b. Oral consent
c. No need for consent

11.qst about Sialometaplasia


12.Contact point incisal/occlusal third
Lower central**************8
Lowers premolars
Lowers molars
13. 65 years old black man wants to have very white teeth in his new denture what should the dentist do:
a- Put the white teeth
b- Show the patient the suitable color first then show him the white one.****************
c- Convince him by showing him other patients photos.
d- Tell him firmly that his teeth color are good
14. If the soft palate falls gradually during recording ,it is easier to record the pps.
if the soft palate falls abruptly during phonetics,it is difficult to record the pps
1.Both statements are true*********************
2.both arefalse
3.first true second false
4.first false second true
15. Pain in mesial canal after ianb in lower molar. Lower molar get additional innervation from which nerve
A.Lingual N
B.Mylohyoid N*****************
C.Ariculotemporal N
D.Mental N
16. What causes blackening of cast which prevents picking
A. Over heating
B. Contamination with gases***********
17. class 2 Kennedy good oral hygiene and free of caries:
A.Circumferential clasp *********************
B.Ring clasp
C.Back action clasp
18. Autoclave mechanism
19. Sharpening curette cutting edge should be at angle
1 50-60
2 70-80************
3 80-90
4 60-70
20. 279. The retainer of rubber dam:
a) Four points of contact 2 buccally and 2 lingually without rocking. **************
b) 4 points of contact 2 buccally and 2 lingually above the height of contour.
c) 4 points of contact 2 mesially and 2 distally.
d) 2 points one buccally and one lingually
21. Pt with endocarditis medical problem in surgery because of
A .bacteremia
B. Septicaemia
C.hypertension
D.mitral stenosis
E.auricular fibrillation
A. A,b,c
B. A,b,d******************8
C. A, d,e
D. B,c,e
E. C,d,e.
22. Low cariogenic food must have
1. Low buffering capacity
2. High mineral content****************
3. High protein content
4. Have pH high
23.child 2 yr old having caries in upper incisors :
baby bottle caries***************
Rampant caries
Nursing caries
24. Forceps to hold tissue for suturing after third molar extraction
1- Stillies forceps. *****************
2- Adson forceps.
3- Curved hemostat
25.mesial and distal root fractured while extracting lower primary second molar.how to manage:
A.Leave it
B.Visualise carefully and remove *************
C.Use dentscan
D.Remove after 1 week
26. Which statement is true about Naocl used in rct
A.Oxidizing effect
B.irrigant of choice**************
C.Better action when diluted
D.Better when combined with alcohol
27. A qstn about ethics
28. Pain during rct even after complete debridement of canal
A.Acute abcess
B.Chronic abcess
C.Chronic apical periodontitis
D.Secondary apical periodontitis*****************
29. Bacteria present in intial caries.
Streptococcus mutans
29. Main purpose of rubber dam in kids
A.Better visualisation
B.For safety*************
C.Improves suction
D.Looks scientific
30. Pt. with active lung TB
A- defer treatment
B- take all precautionary measures and treat pt
C- emergency treatment done with maximum precaution *******************
D. No treatment
31. Minimum length of GP to seal apical part when u place post
1- 4 to 5 mm****************
2- 2 to 3mm
3- 5 to 10mm
32.8 yr old kid with discoloured upper incisor, had trauma 9 months ago with no change in radio graph or
pain.treatment:
A. No treatment***************
B.caoh pulpotomy
C.DPC
D.pulp extripation and apexifixation
33. To enhance strength properties of ceramometal restoration, it is important to:
a. Avoid sharp or acute angles in the metal structure.
b. Build up thick layer of porcelain.
c. Porcelain should be of uniform thickness and any defect of the preparation should be compensated by the
metal substructure.
d. Compensate any defect in the preparation equally by porcelain and metal substructure.
e. a and b are correct.
f. a andc are correct. ****************
g. b and d are correct.
34. pt came to ur clinic for review of cd, no complaint during speech or swallowing.
Lips looks elongated, narrow vermillion border:
a) vitamin b deficiency
b) decreased vd
c) malpositioned upper teeth***************

Exam23
1. Flash sterilization.132 degree for 2-3 minutes
2. 50 yr old female have paroxysmal unilateral short pain which aggravates on touching
Atypical odontolgia
Atypical neuralgia
Trigeminal neuralgia***************
3. Topical fluoride varnish for office use
Act
Prevident*************
Some phasphat
4. Retractor used to retract cheek and flap
Mennosota
5. 8 yr old patient has primary lower canine space infection what is the management
Extract canine without space maintainer*****************
Extract canine and band and loop
Extract both the canine without SM
Extract both the canine with lingual arch holding bar
6. Examiner checked a second yr students RPD which has not a main component that dissipate the occlusal
forces to tissue surface
Rest**************
Direct retainer
Indirect retainer
7. Bilateral edentulous area located posterior to remaining natural teeth
Kennedy I***************
Kennedy II
Kennedy III
Kennedy IV
8. How many direct retainer present in kennedy applegate rule I
6
2***************
3
4
9. Guiding plane for the partial denture
Cervical
Occlusal
Distal****************
Buccal
10. Old patient who is 80kg wat s the max LA with xylocaine 2% 1:100000
365
560**************
11. Disadvantage of full thickness flap
Delayed secondary healing
Scare tissue formation****************
12. Tertiary dentine is
Calcified dentinal tubules
Irregular/ secondary dentine*****************
Smthing else wrthere. Sorry dont remember
13. Lab report of a 60 yr old patient who need to go for surgical extraction has atypical plasma
cholinesterace enzyme in blood. Choice of LA
Prilocaine hcl with adrenaline 1:200,000***
Prilocaine HCL without adrenaline
Procaine with adrenaline
Procaine HCL without adrenaline
14. How to increase the image quality
Increase the distance between object and cone**************
Decrease the distance between object and cone
Place the film perpendicular to the long axis of the object
Place the cone perpendicular to the long axis of the object.
15. A pic of Missing bilateral maxillary canine. X ray shows radiolucency around the crown of impacted
teeth. What s the diagnosis
Dentigerous cyst***
Nasopalatine cyst
Ameloblastoma
16. 10 yr old boy with slow growing bilateral mandible. Xray shows radiolucency intermingled with
radioopaque
Ameloblastoma
Cherubism**************
Cementoma
17. A cyst which has clear golden yellow fluid on aspiration
Aneurismal bone cyst
Dentigerous cyst***
Staffnes cyst
18. Root coverage technique for class ii recession (lazan some technique name. not sure)
Partial thickness flap with split
Partial thickness flap without split
Connective tissue flap without split
Connective tissue flap with partial split*****************
19. Fluorosis commonly seen in
Lower Ist molar
2nd molar and premolars of maxilla**************
Upper Lateral incisor
Lower central incisors
20. Hemidesmosome Basel cell epith. attached to:
A. Lamina Lucida**************
B. Lamina densa
C. Lamina propria.
21. What make priority to private clinic than community dental clinic
Need for assessment*************8
Insurance
22. A pic of White chalky appearance on the lateral incisor of orthoband removed pt who has moderate
caries risk
Enamel hypoplasia
Incipient caries***************
Dentinogenisis imperfect
23. A pt had immdiete denture. He comes after 6 months and complains of pain in the lower front region. On
examination pt has inflammation and ulcer on the labial surface. What is the diagnosis
Eppulis fissuratum****************
Hypertrophic labial frenum
Laceration in the papilla
24. Fluoride toxicity in dose in milligram for adult patient
32-64mg per kg
25. Arrange in descending order based on importance
Parallelism, height, occlusion, retention*********************
Height, occlusion, parallelism, retention
Retention, occlusion, parallelism, height
Parallelism, occlusion, retention, height
26. Bilateral FPD patient class III occlusion
Balanced occlusion
Unilateral occlusion**************
Mutual occlusion
27. Impression material in which u can pour two times
Poly ether
Polysulphide
Additional silicone***************
Alginate
28. What is the method of removal of alginate impression from the patient mouth in order to maintain the
recorded tissue surface from wear and tear
Remove gently
Remove with snap**************
Remove forcefully
29. Traumatically intruded upper central and lateral incisor which has open apex. Wat s the management
Observation*******************
Extraction
Pulpectomy
Pulpotmy and calcium hydroxide
30. There is an external resorption in the central incisor of a patient who went for orthodontic treatment.
Management
Extraction
Pulpectomy and GP
Calcium hydroxide at the resorptive site***
31. Fracture and intrusion of the central incisors in children are mainly due to
Protruded***
Retruded
supraerupted
32. Prognathic maxilla with central incisors are linual than lateral incisor. Angles classification
Clas I
Class II div I
Class II div II *******************
Class III
33. Management of mucocele
Incision
Excision***
34. Instrument used to remove the broken instrument
Schwartz Peizo retriever
35. Ferrule effect by
Forceful extraction
Orthodontic extrusion***
36. Minimum width of the remaining tooth structure around the post and core in millimeter
3
2
1***
37. Gingival hyperplasia commonly seen in
Phenytoin (dilantin)***
Calcium channel blocker
38. Patient jumps on percussion related to vital tooth
Symptomatic apical periodontitis***
Asymptomatic apical periodontitis
Irreversible pulpitis
39. Mild fluorosis
Micro abrasion***************
Home bleeching
Veneer
40. Chair side disinfectant for GP
H2O2
5.2%sodium hupochloride***
Saline
41. Why primary dentition does not affected by fluorosis. (Question was long not this simple)
Placenta doesnt cross fluoride
42. Which is common among these
Cleft lip
Cleft palate
Cleft lip and palate***
43. Carbohydrate digestion starts at
Mouth***
Fundus
Duodenum
Intestine
44. Indication of preventive resin restoration
Discoloured pit and fissue with shallow caries***
Same with deep caries
Proximal caries
45. Most colour stability in
Porcelain***
Acrylic
Composite
46. Moisture contamination may results in
Change of the colour***
Increase compressive strength
47. Floss used to
Remove calculus
Remove plaque***
Remove overhangs
48. When should start brushing
At the age of 2
When first tooth erupt***
When starts going to school
49. Calculus is
Soft debris
Calcified plaque***
50. When u condense the amalgam after trituration
Immediately************
After 3 min
Condense vertically and laterally
51. Hemophilia B clotting factor
Clotting factor VIII
Clotting Factor IX***
52. Extra canal in lower canine seen in
Buccal
Lingual***
Mesial
Distal
53. Using antibiotics for carditis pt
Ortho band removal
Intraligamentary anaesthesia**************
Suture removal
54. Zoster caused by
Herpes simplex zoster***
Human papilloma
55. Exact term of MTA- mineral trioxide aggregate
56. Polycystic lateral periodontal cyst is
Botroid cyst***
Aneurismal bone cyst
57. Best method to reduce bacterial load before sterilization
Ultrasonic cleaner***
Disinfectant
Soap
UV
58. Development of mandible from which cartilage lateral from pharyngeal arches
Meckels cartilage
59. Distance between the person and the ceph in feet
6
5***
4
10
60. Anticaries sugar substitute
Glucose
Xylitol***
Sucrose
61. Extraoral block after touch pterygoid plate move
Posterior superior***
Posterior inferior
Antero superior
Anteroinferior
62. What right about gracey and universal curette
Gracey has one cutting edge where as universal has two
Gracey is for specific area where as universal entire mouth
Gracey has half circle cross section where as universal triangular
Gracey have bi beveled edge 70 degree universal have beveled edge 80degree
A+D
A+B**
A+B+C
A+B+C+D
63. Cross section of K Reamer
Circular
Square
Triangular***
64. Dentin contains which type of collagen fibre
Type I***
Type II
Type III
Type IV
65. A pic of Verruciform Xanthoma. Caused by
Virus
Immunosuppression***************
Trauma
Allergy
66. Acute injury to tissue during improper flossing
Laceration of interdental papilla****
Recession
PD injury
67. Main use of dental floss
Remove calculus
Remove overhang
Remove interproximal plaque****
Remove food debris
68. Mesial and distal root fractured while extracting lower primary second molar. How to manage:
Leave it
Visualize carefully and remove****
Use dentscan
Remove after 1 week
69. Minimal duration from first RCT to reendo
I months************
2 wks
Six days
2 months
70. 3rd point in a facebow
Occlusion
Orbital
Nasion***********
71. Patient with 5 years old denture has a severe gag reflex, upon history he says he had the same symptoms
in the first few days of the denture delivery and it went all alone:
Patient has severe gag reflex.
Patient has underlying systemic condition***
Denture is overextended
72. Neonatal tooth means
Before birth
0-30 days after birth***
6 months
6 years
73. 6 yr old patient with thumb sucking habits. Management
Early appliance***
Psychological counceling**************
Exam25
1)Best way to disinfect impression
A.Autoclave
B.Uv chamber
C.Disinfectant
D.Chemical sterilization***
2)Most biocompatible base to pulp and periodontal tissues
A.Zinc polycarboxylate ***
B.Zinc phosphate
C.Calcium hydroxide
D.Gic
3)While u are removing impression from a cast u need to be careful not to tear it which impression material
A.Polyether
B.Condesation silicone************
C.Addition silicone
D.Polysulfide
4)Optimum level of Flouride in water in ppm
A.10
B.1.0***
C.0.1
5) A qs about Treacher collins syndrome
6)Congenital syphilis
7)Hyperparathyroidism (scenerio of a woman with renal stones n all)
8)Suture after extraction of all teeth in arch
A.Interrupted ***
B.Figure 8
(There was no option for continuous that means interrupted is the right choice)
9)What does dolicocephalic means
A.Long skull***
B.Short skull
C.Long face
D.Short face
10)Gates glidden no 1 diameter..50
11)Mandibular Canine impaction which radiograph
A.Occlusal***
B.Periapical
C.Panorama
D.Cephalograph
12)Canine miler class II recession which flap
A.Full thickness with full split
B.Full thickness with partial split
C.Connective tissue graft with full split
D.Connective tissue graft with partial split***
13)Force of dislodgment of denture
A.Force of adhesion
B.Adhesiveness of food*************
C.Force of gravity
14)Why packing of instruments before sterilization
A.To maintain sterility***
B.To prevent corrosion
C.For labelling
D.For easy transfer
15)Trauma to maxillary central n u did pulpotomy success of treatment depends upon
A.Tooth becomes Asymptomatic
B.Root continues to develop***
16)Stainless steel crown in MOD in a child
17)8 years old Missing right primary molar missing left primary canine 1 st primary molarn second prjmary
molar..space maintainer
A..Fixed Lingual arch***
B..Bilateral Band n loop
C..Distal shoe
D..Hawley appliance
18)Centric relation is
A.Bone to bone
19)Implant osseointegration
A.Fibrous
B.Bone ***
C.Epithelial
20)How many percentage of proximal dentine caries are not cavitated
A.10
B.20
C.30
D.60****
21)Lateral Condylar guidance hanaus formula..
A.L=H+12/8
B.L=H/12+8
C.L=H+8\12
D.L=H/8+12***
22)Old patient wants complete denture Flat ridge teeth must be set at which angle
A.0 degree***
B.20
C.30
23)Preformed aesthetic crown which material aesthetic
A.Polycarbonate ***
B.Gic
C.Composite
D.Resin
24)Acid base reaction which material..GIC
25)Effect of water on restoration
A.Change in color of restoration***
B.Increase strength of restoration
C.Soft resin
D.Decreases weight of restoration
26)A patient having amalgam filling in 26 n now there is recurrent caries whats the treatment
A.Remove caries n make new restoration***
B.RCT n restore
27)Best material for perforation
A.Mineral trioxide aggregate***
B.Mineral tetra oxide aggregate
C.Material trioxide aggregate
D.Material tetra oxide Aggregate(option with just similar names to confuse)
28)Which surface of distal root mostly perforated
A.mesial
29)Dog bite qs as in files
30)A 22 year old sudden swelling of one side of face , there is also condyler hyperplasia, severe
malocclusion n spaces in teeth
N partial macroglossia
A.Facial hemihypertrophy*************
*************
B.Acromegaly
C.Partial agnathia
D.Facial hemiatrophy
31)Implant question pic showing impalnt coping in impression n some flowable material is placed around it
with a syringe whats is this for
A.Implant mask
B.Gingival mask****
Rest 2 options dont remember
32)Maxillary molar impaction how to prevent tearing ofo flap
A.Sufficient flap raised***
B.Inverted u shape flap
C.Retracting flap forcibly
D.Including greater petrosal artery in the flap
33)Rideal walker test for?
Disinfectant
34)Temperament is a natural emotion n may have a gentic factor
What best decribes it (i dont know what was it )
A.Unstability over time and late appearence
B.Sability over time and early appearance
C.Moderate stability over time and early appearance**************
appearance
D.Moderate stability and late appearnce
35)Drug causing hyperplasia of gums
Phenytoin
Rest 3 names were analgesics so it was clear
36)After gingivectomy surface epithelisation occurs in
A.3 days
B.5-14***
C.14-21
D.Over a month
36)Some curve name n asked which cells mature first
A.Neural*
B.Genital
C.Lymphoid
38)Microorganism which can invade epithelium
A.Porphyromanas gingivalis***
B.Fusobecterium
C.Capnocytophega
39)What is true about perakeratin
A.Not found in gingiva
B.Found in oral mucosa
C.Cells are retained in keratin layer***
D.Keratinocytes granules are present
40)Rubber dam qs
A.Plastic retainer to be used for teeth with recessed pulp chamber
B.Retainer having four point contact on tooth****
C.Plastic retainer give best retention on teeth with less coronal structure
41)For Which restoration we remove caries with hand instrument only
A.Pit and fissure sealent
B.Composite restoration
C.Preventive restoration
D.Alternative technique restoration***
42)Which teeth difficult to anesthetize
A.Mandibular molars****
B.Maxillary molar
C.Maxillary anterior
D.mandibular anterior
43)When pt cant locate the jaw n we do anaesthesia test from where to start
A.Right maxiallry posterior***
B.Right mandibar posterior
C.Left maxillary posterior
D.Left mandibular posterior
44)A tooth with RCT and is asymptomatic but there is Radiolucency 3mm n have to do re endo but
Also we want intracanal free of microbes..how to achieve
A.Coronal flare
B.Debritement till apex****
C.Leaving tooth open
D.Apicectomy after re endo
45)A scenerio qs on acute exacerbation of chronic abcess
46)Reversible pulpitis scenerio
47)Lateral periodontal cyst scenerio
48)Condensing osteitis scenerio and radiographic features
49)There is little space between max central incisors in a 9 years old n there is no deep bite what u do
A.Functional appliance
B.Fixed ortho appliance
C.Removable ortho
D.Tell pt there is no need for treatment ***
50)After surgery dentist removes gloves washes hand n enters record in the register and then proceeds to
autoclaving instruments,what will he do first
A.put on heavy utility gloves***
B.Removes personal protection
C.Put in autoclave
D.Rinse n brush instruments
51)After patient is seated ,his eyes closed ,he is prepped n drepped what u do now
A.Do oral examination ****
B.Give Prohylaxis
C.Take Necessary radiographs
D.Give oral hygiene instructions
52)Bacteria for infective endocarditis
A.Staph aureus***
B.Stre. Virridans
C.Actinomyces
53)Step deformity after fracture in mandible is due to
A.Upward pull of masseter n temporalis***
B.Forward pull of medial pterygoid
C.Downward pull of geniohyoid
D. pull of lateral ptrygoid
54)Oral manifestations of HIV in a child
A.parotitis
B.kaposis sarcoma
C.herpes gingivostomatitis****
55)Scenario for decreased vertical dimension
56)A question related to Angular stomatis
57)Why we put Reverse bevel in cast metal inlay
A.For retention****
B.For resistance
58)Pic of lingual tori
59)What medium to put avulsed tooth
A.Saliva
B.Water
C.Cold milk****
D.Milk
60)Dental floss used for: interproximal plaque
61)Plaque color complex in 18 year..purple
62)Multiple caries present n at high risk for caries treatment management
A.Permanent restoration n oral hygiene instructions
B.Oral hygiene instruction n then permanent restoration***
C.Temporary restoration n later permanent
63)A lady taking hypertensive drugs n presents with dry mouth n dry eyes but no pain in joints
A.Xerostomia
B.Primary sjogern***
C.Secondary sjogern
64)Which Graft is best for osseous defect
A.Freeze dried
B.Cortical decalcified freeze dried
C.Cancellous decalcified freeze dried************
D.All have same potential
65)Bacteria responsible for initiation and progression of caries..strep mutans and lactobacilli
66)Scaling in epileptic motion?
A..ultrasonis
B.Piezoelecrtic
C..Magnetistrictive****
D.scaler
67) A qs about Kennedy class II
68)PERICORONITIS
A.Inflammation around erupting 3 rd molar ****
B.Swelling n suppuration
C.sensitivity to Hot n cold
69)Dose for systemic flouride depends upon..Age
70)Which of these make floor of mouth
A.mylohoid***
B.geniohyoid
C.infarahyoid
D.diagastric
71)Acess cavity shape in max lateral incisor with recessed pulp
A.oval****
B.triangular
72)Disinfectanat that causes rusting of instruments
A.Hydrogen peroxide
B.Sodium hypochlorite****
C.Idophors
D.some aldehyde name
73)prilocaine4% 1/200000 cartridge color
A.yellow***
B.red
C.green
D.blue
74)A question about rongeur forceps
75)After restoration there is some extra material how to remove
A.knife
B.burnisher
C.carver****
D.scraper
76)Parts of dental elevator
A.tip shank handle
B.blade shank handle****
C.tip hinge handle
D.blade hinge handle
77)Cells responsible for pain
Free nerve endings ****
Meisner corpuscles
78)Pseudomanas aerogenosa
79)Irrigant..lowering surface tension
80)Prophylactic antibiotic
..placement of orthodontic bands
81)Composite efficient polymerisation
82)all symptoms of herpes simples jn a child( ulcers,fever,malaise,loss of appetite

Exam26
1. Nerve sensation to TMJ?
a. Auriculotemporal nerve
2. Muscle palpated at the temple area
a. Temporalis
3. Type of bone best suitable for implants
a. Type I
b. Type II***
c. Type III
d. Type IV
4. Most commonly used blade in Oral surgery
a. 15
5. Management of root that got displaced into the maxillary sinus
a. Leave and observation
b. Administer antibiotic
c. Remove it as soon as possible***
6. What interferes with wound healing
a. Loose sutures
b. Necrotic tissue***
7. How many carpules can you give from lidocaine 2% to a child who weighs 20 kg
a. 2.4*******
b. 3.8
8. In which case you cant give epinephrine
a. Hyperthyroidism
9. You give Posterior superior alveolar nerve block , you are working on the first molar and the patient still
has pain what do you do?
a. Give buccal infiltration
10. Comminuted mandibular fracture , whats the name of the technique for external fixation?
Clark technique**********
11. Suspecting that right condyle is shifted mesially , which x-ray
a. Occipitomental
b. Reverse towns***
c. Lateral oblique with 30 degrees
d. PA for mandible
12. 7 year-old child , you want to do radiographic survey
a. 12 periapicals, 2 bitewing
b. 2periapical, 2 bitewing, 2 occlusal*****************
c. OPG and 2 occlusal
13. Best imaging for TMJ disc perforation:
a. Arthrography
14. Respiratory flow rate (L/min) in children?
a. 1-5?
b. 5-9***
c. 10-14
15. Calculate mean arterial pressure?, Where systolic is 140 , Diastolic 80
a. 70
b. 80
c. 90
d. 100***
16. Cells that dont regenerate unless theyre injured
a. Neural cells***
b. Blood cells
17. Which antibiotic inhibit cell-wall synthesis
a. Tetracycline
b. Penicillin***
18. After root fracture , repair is evident how?
a. Presence of calcified tissue next to the root
b. Connective tissue
c. interproximal bone and connective tissue****************
19. True about odontoblastic layer?
a. Filled with odontoblastic processes***
b. Charectarised by cell-to-cell gap junction
c. Charectarised by cell-to-cell tight junction
d. Have higher concentration of cells?
20. Allergy from Methyl Methacrylate used in dental laboratory results in
a. Contact dermatitis***
b. Type I hypersensitivity reaction
c. Lung irritation and respiratory problems
21. What spreads Airborne pathogens the most
a. Droplets
b. Aerosols***
22. Growth of the maxilla is primary by?
a. Intramembranous ossification***
b. Endochondrial Ossification
c. Both
d. Cartilage
23. Pt with notched incisors, turns out he has congenital Syphilis, at which stage of enamel formation did this
defect happen?
a. Apposition
b. Morphodifferentiation***
c. Maturation
24. What causes hyperalgesia
a. Prostaglandin and serotonin?***
b. Substance P
25. Most commonly used intracanal medicament:
a. Ca(OH)2***
b. EDTA
26. Diffuse facial swelling, #37 is necrotic with large periapical radiolucency, your diagnosis:
a. Necrotic pulp acute abscess
b. Necrotic pulp with symptomatic apical periodontitis****
27. Cast post and core is best used in which canal?
a. Flared?****
b. Straight
c. Curved
28. Tooth cusp is broken, dentine exposed and patient has brief sharp pain, whats the pulpal status?
a. Reversible pulpitis***
b. Irreversible pulpitis
c. Activated A fibers
d. Normal pulp
29. RCT should end at:
a. Minor apical constriction****
b. Radiographic apex
c. Anatomic apex
d. Major apical constriction
30. What is the advantage of rotary instrumentation over conventional
a. Faster***
b. Better cleaning
c. Better shaping
31. The 2% taper in files mean?
a. Taper from tip of the file to D16
32. Patient that is diabetic and insulin dependent presented to the clinic with swelling related to a tooth,
penicillin allergy, what will you do?
a. Start RCT then prescribe Clarithromycin
b. Give antibiotic then start RCT***
33. Lower canine when it has a second canal, where is its location in relation to the main canal?
a. Mesial
b. Distal
c. Buccal
d. Lingual****
34. mild yellowish brown stains. H/o of teracycline medication in chilhood. which is the best method to
removed those stains?
a. Microabrasion with Pumice + Hcl
b. Home bleaching****
c. Composite veneers
35. Most common feature in diabetics
a. Gum abscess
b. Gingival recession****
36. Contraindication for gingivectomy
a. Supragingival pocket and bone defect
37. Smoker for the past 15-20 years with chronic periodontitis, signs and symptoms of inflammation:
a. More than non-smokers
b. Less than non-smokers***
c. Same as past-smokers
d. Same as non-smokers
38. Area specific periodontal instrument? .. gracey curettes
39. How will you asses oral hygiene / efficiency of oral hygiene methods prescribed to the patient?
a. Plaque index****
b. Calculus index
40. Best Perio graft graft material for 2 wall bony defect
a. Freeze-dried bone
b. Decalcified cortical freeze-dried bone
c. Decalcified Cancellous freeze-dried bone*************
d. All materials have same result
41. Acute sign of aggressive flossing
a. Gingival recession
b. Lacerated interdental papilla***
42. Centric relation is?
a. Bone-bone relation***
b. Bone-tooth
43. in Hanaus Quint , when you increase incisal guidance what happens to condylar guidance?
a. Increase
b. Decrease
c. Same***
44. Patient with missing lower incisors, what is Kennedy classification?
a. Class IV
45. What is the most common problem with Ackermans type of clasp
a. Mobility
46. Patient had his denture done 3 times before, not satisfied and wants a new one, you checked the previous
dentures and theyre good nothing wrong, what type of patient
a. Philosophical
b. Exacting**************
c. Indifferent
47. Denture Pt comes with gag and discomfort and when you press on the midpalatal region, bubbling from
posterior margin of the upper denture, what to do?
a. Trim denture
b. Fluid wax impression****
48. Patient came few weeks after partial denture is placed Kennedy class I complaining of pain in the area of
the denture, and gum bleeding I think, what is the cause
a. gum stripping design***= gum stripping means RPD without occlusal rest
49. You want to take impression after crown preparation for upper premolar, the clinical crown is short so
you prepare the margins to be subgingival, the margins is very deep , theyre close to epithelial attachment ,
what will you do in order to register the finish line
a. Place retraction cord****
b. Electrosurgery
50. 65 year old patient came to the clinic, he had tooth no. 17, 33, 34 and 43 present in the mouth, all had
Grade 1 mobility except 17. 17 was carious and highly mobile. Treatment:
a. Extract all the teeth and give full dentures
b. extract 17 with maxillary complete denture, and over denture in lower
c. extract 17 and 34 and give over denture in lower and maxillary complete denture***
51. Patient with missing upper lateral , Central and canine intact
a. Implant***
b. Bride
c. Resin-bonded bridge
52. Pt presented worried about esthetics, has missing upper lateral, the upper central is labially inclined, Both
central and canine has proximal and palatal decay, How will you restore?
a. Resin bonded bridge
b. Implant
c. Labial veneers with palatal composite
d. Conventional fixed bridge************

Exam28
1)FPD in mandibular molar , the preparation is short of lack
of retention , we want to do provisional crown using zinc
oxide eugenol cement what to do :
a. Thick , creamy mix of the cement***
b. Add petroleum to the cement
c. Maintain dry field until cement set
d. Remove hardened part of cement in interproximal
area by using explore
2: Gingival margin trimmer used in
A. Finishing class5
B. Beveling class 2***
3: Which contraindicated with sickle cell anemia
A. Aspirin ***
B. Acetaminophen
C. Local anesthetic containing vasoconstrictor
4:Last step in the insertion of PFM crown?
a Glazing of porcelain
b Polishing of metal****************
c staining of porcelain
d contouring
5: When using buccal object rule where will lingual appear if we move cone mesialy
A distally
B. Mesialy***
C. Buccally
D.lingualy
6: Distance between two implants
2mm
3mm***
4mm
5mm
7:Cavosurface angle for amalgam
60
90***
70
110
8: Optimum crown to root ratio for fpd
1:2
2:3***
3:2
1:1
9: 8years Patient came to your clinic has impaired hearing, upon examination his mouth you found copper
color lesion, notched incisor and mass on the occlusal surface of the molars. This patient has:
a) Gardner syndrome.
b) Congenital syphilis. *****
C turner hypoplasia
D Gorlin Goltz syndrome
10: At which age parent should go to the
orthodontist for consultation:
1. When all permanent teeth erupt
2.when permanent centrals erupt***
3. when there is spacing present
4. when there is crowding present
11: Zinc polycarboxylate cement is better than zinc phosphate cement in:
a- Compressive strength.
b- Low solubility.
c- Film thickness.
d- More biocompatible ****
12: Root perforation is treated by:
1. MTA. ****
2. Ca (OH) 2.
3. Root canal with GP
.
13: The adverse effect of orthodontic treatment commonyl seen after treatment
A enamel hypocalcification***
B root resorption
C tooth relaps
14: After trauma tooth is partial displaced coronally injury is
A:intrusion
B:extrusion************8
C Subluxation
15: Allergy from Methyl Methacrylate used in dental laboratory results in
a. Contact dermatitis**************
b. Type I hypersensitivity reaction
c. Lung irritation and respiratory problems
D eye skin and mucusal irritation
16: During final inlay cementation which of the following you will do?
A. Polishing
B. Remove occlusal interferences
C. Lowering occlusal surface.
D. Burnishing of peripheries of restoration **************
.
17: Old pt. has some upper molars overerupted, While lower jaw anterior has gingival recession, degree of
mobility & Kennedy Cl 1:
a. acrylic RPD with occlusal adjustment of upper over erupted teeth
b. swing lock also with adjustment***
c. over denture
18: Pt came for prosthetic treatment , she has missing #24 #26#27#28 and #25 is left as peir abutment , it has
lingodistal inclination , Mesiooccusal amalgam restoration and grade II mobility, what is ur treatment?
A-Extract #25 and RPD****************
B-FPD on 23-25 and RPD on 26 27 28
C-Overdenture RPD on 25
D-RPD
19: In complete dentures denture adhesive usually requires for which typenof palatal valut
A u shaped valut
B flat*****************
C v shaped
D ovoid
20: The degree of taper of crown prep. :
A. 3-5
B. 15
C. 6***
21: Type of orthodontic force applied to teeth with compromised attachment level:
a- tipping
b- light***
c- heavy
d- intermittent
22: Child has mild Tetracycline discoloration in
permanent tooth what is the proper treatment:
1. Composite veneer
2. Home bleaching***
3. Pumice micro abrasion
4. Porcelain veneer
23: Hyperplasia of nerve fiber
Calcitonin...
Prostaglandin + serotonin ****
Lysosomes enzyme
Substance p...
23: When first expose child to use dental brush..?
A. Eruption their primary teeth***
B. At age of 2 y
C. At age of 5 y
D. When he entered primary school
25: Dental materials are classified as
A. ceramic, polymers and composite
B. ceramic, polymers and alginate.
C. ceramic, polymers. Cement
D. ceramics, metals, polymers,composites****
26: In primary tooth for restoration before putting the filling u put:
a. base.
b. calcium hydroxide.
c. varnish.
d. you put the filling after proper cleaning and drying.***
27:After usage of sharp scalpels, needles what's the best management:
1) throw in a special container of sharp instruments. ***
( Sharp container ).
2) sterilize and reuse.
3) through in ordinary plastic waste basket.
28: The sequence in deep carious lesion close to pulp are
a. GI base, varnish, caoh2
b. Varnish, GI base, caoh2
c. caoh2, GI base, varnish****
d. caoh2, varnish, GI base
29: Patient come with complaint of loose denture when press Complete Denture, it bubbles in the posterior
part near the
palatal seal, and good retention in the anterior Vestibule and Buccal vestibule, what is the problem:
A) Over post dam
B) Under post dam+++
C) Over extended
D) Under extended
30: Brushing technique in which sides of bristles takes active part
Modified stillman***
Charters..
Bass
31:Patient complains from pain in TMJ. During examination you noticed that during opening of the
mouth mandible is deviate to the right side with left extruded. Diagnosis is:
a) Condylar displacement with reduction.
B) Condylar displacement without reduction. ***
C rheumatoid arthritis
32: the CBCT have the following property
a- best to show TMJ disk***
b- expose the patient to large amount of x-ray
c- use for routine radiographic examination
33: patient with interproximal caries detected by bitewing radiography, you want to detect caries that reach
the pulp, what to use:
a-periapical****
b-bitewing with another angulation.
C occlusal
D Panorama
34: The water rins devices for periodontal therapy has a main goal which is:
a- remove plaque
b- prevent plaque attachment
c- dilute bacterial toxin***************
d- remove dental pocket
33: Edentulous pt. class II kenndy classification 2nd premolar used as abutment when we serving
we found mesial undercut what is the proper clasp used:
1/ wrought wire with round cross section. ***
2/ wrought wire with half round cross section.
3/ cast clasp with round cross section.
4/ cast clasp with half cross section
34: Zirconium post has:
a-high compressive strength and low tensile str.
B- low compressive str. And low tensile strength
c-High compressive str. And high tensile str.*****
D- Low comp low tensile
35: What is meant by hyper-apnea
A increase in rate and depth of breathing ***
B increase in rate of breathing
C increase in depth of breathing
D kussmaul breathing
36: Most commonly used blade use in oral surgeries
Blade # 11
Blade # 12
Blade # 10
Blade # 15****
37: most important biochemical consideration in implant:
a. abutment is made from titanium alloy.****
b.abutment be in 2 parts
38: Most biocompatible alloy when used intra oraly
A cr-co alloy
B titanium alloy***
C nikl chromium alloy
C cast metal alloy
39: 18 years old pt. The bacterial complex present in his mouth is:
A. Red complex
B. Green complex
C. Purple complex****
40: 18 year old patient have unilateral swelling of mandible which is painful when palpate
And there is x ray pic shows radiolucency around the crown of impacted lower thrid molar
Dentigerous cyst***
Ameloblastoma
Okc
Osteosarcoma
41: Young female patient with skeletal class 1 there is spacing present in anterior teeth. Her mother also have
mild spacing due to missing upper lateral
What is cause of spacing in patient
A hypodontia***********
B avulsion
C congenital
D dileceration
42: Access opening of maxillary lateral with recessed pulp
A triangular
B oval****
C rhomboid
43: Access opening of mandibular molar
A Rhomboid****
B round triangular with base towards buccal
C round triangular with base towards distal
44: Floor of mouth is mainly formed by
A diagestric
B genioglosus
C Mylohyoid****
D stylohyoid
45: patient came to you with multiple vesicles on the attached gingiva in the area of upper anteriors after
having extensive dental treatment the day before, what is the possible cause of the ulcers?
a. recurrent aphthous ulcer
b. recurrent herpes gingivostomatitis***
c. allergic stomatitis
d. mucositis
46:In FPD saturation of clour what does it means
A chroma****
B value
C hue
47: The percentage of simple caries located in the outer wall of the dentin (proximal sides of the tooth) which
left without cavitations is around:
A 60%.**************
B 40%
C 10 %
D 50%
48: Preventive resin restoration PRR is mainly used to prevent
A initial caries lesion
B caries progression****************
C
D
49: Organism which cuases initiation and progression of caries
Streptococcus mutans and lectobacilus
Xray picture of cliedocranial dysplasia
50: best describes caries progression?
A. Cant be reversed
B. demineralization without remineralization
C. Demineralization with remineralization**************
D can be arrested
51: foramen for accessory canal due to defect
A. Dental sac
B. Hertwig sheath***
C. Dentine matrix
52: Most favourable habitat for streptococcus mutans
A pit and fissure****
B buccal
C lingual
D proximal
53: pa ent smoking 15 -20 cigarettes a day for 8 years he com with
complain
of pain in gingival Diagnosed as chronic gingivitis
a-gum shows inflammation as former smoker
b-less intensity than unsmoker****
c-more intensity than non-smoker
d-same as non-smoker
54: A 21 year old man comes to the dental clinic. You notice he has plaque amd calculus accumulation. A
SCaler with elliptical motion was used in this patient for scaling. Which type of scaler ?
A Piezoelectric
B MAgnetostrictive,***
C Sonic,
D Ultrasonic
56: Which causes more spread of air born infection
A splater
B droplets
C spills
D aerosols***
56: The remnants are rest cells of srres of
A hertwig root sheath
B vestibular lamina
C dental lamina***
D oral epithelium
57: Anesthetic testing (intraligamentery injection)is most effective in localizing pain to which of the
following?
Specific tooth****************
Mandible or maxilla
Across the midline of the face
Posterior tooth
58: which of these is used subgingival?
hoe
hachette
gracey curette***
chisel
59: Main advantage of glass ionomer to prevent caries
A release fluoride***
B less soluble
C wear resistance
D aesthetic
60: Preparation of cavity for composite
A depends on caries****
B should be .5 mm in dentine
C sould be in .5 mm in enaml
61: Gates gliden
A have numbers 3 to 9
B use to prepare coronal part of canal***
C breaks when bind in canal
D made of nikle titanium
62: Patient had anaphylactic shock due to penicillin injection, what's the most important in the emergency
treatment to do:
a. 200 mg hydrocortisone intravenous.
b. 0.5 mg epinephrine of 1/10000 intravenous.
c. adrenaline of 1/1000 intramuscular.***
63:Mentally ill child,the best way to apply fluoride:
a. Acidulated phosphate fluoride.
b. Natural sodium fluoride.
c. Fluoride varnish. ***
D. Stannous fluoride
64: 28 years old female is advised to brush her teeth forcefully to prevent calculus
why?
A. Break plaque layer***
B break caculus
C
D
65: blood ,suction fluid and liquid waste should be disposed
A in rigid container
B in drain which is connected to sewerage ***
C in plastic bags
66: 6 year old child presents with bilateral facial swelling in ramus x ray shows well defined multi location
and molars are anteriorly displaced
A fibrous displasia
B cherubism***
C cementoblastoma
67: main reason for unilateral cleft lip
A perforation of buccopharyngeal membrane
B failure to fuse medial nasal process with maxillary process**************
C failure to fuse lateral nasal process with maxillary process
68:65 year old patient presented with pain on biting in left lower posterior teeth 47 have temporary root canal
since 6 months and 46 have amalgam fillinf since 2 years. There is 5-6mm deep isolated pocket on lingual
and buccal side of 47 with other wise healthy periodontal tooth
Diagnosis of pain
A Micro leakage in 47
B short root canal filling in 47
C vertical root fracture*****************
D micro leakage in 46
.
69: Re treatment
if endodontic treated tooth operator not reach to the all length
no stick filling
A.Fracture instrument
B.Mud
C.Ledge***
D perforation
70: Fracture in neck of mandible which nerve affected
Infer alveolar
Lingual
Mental
Auriculotemporal****
71: Rubber dam qs
A.Plastic retainer to be used for teeth with recessed pulp chamber
B.Retainer having four point contact on tooth****
C.Plastic retainer give best retention on teeth with less coronal structure
72:Which of the following may cause gingival enlargement :
1. Phenytoin ( Dilantin)***
2. Cyclosporine
3. Nifedipine
4.Aspirin

Exam30

sucrose ?What is the sugar that is metabolized by bacteria and cause demineralization -1
with examination of ,difficulty in swallowing ,years old patient who has iron deficiency anemia 21 -2
: you found ,barium sulphate
geographical tongue .a
burning mouth syndrome .b
** plummer vinson syndrome .c
diabetic patient .d
: of flourid in %90body retain -3
kidney -a
blood plasma -b
**calcified tissue -c
?anterior teeth in a child is the punch hole in rubber dam for -4
***asmall
medium(b
large(c
the hole size isnt important(d
:Sectional cast post used in which tooth -5
insisors-A
canines-B
premolars-C
***molars-D
CUT THE SEPTA ?FIRST THING U DO WHEN REMOVING RUBBER DAM -6
Standardized gp the same diameter and size ?zed GP conventional Gutta Percha comparing with standardi -7
as file
XEROSTOMIA ?;Common feature find on Uncontrolled diabetic pt . -8
? Question about Preparation rest seat before impression was evaluatedd -9
design allow rotation movement_a
icknessb cut uniform th
??c Sorry I do not remember
Picture about vertical fracture -10
Picture about shift in midline -11
? Question about the using fpd to support teeth in which case -12
fracture -a
recurrent caries -b
discolleration -c
****************periodontal desease -d
? There was long Scenario about pt then he said that pt has ameloplstoma and asked about tt -13
***A part resection with wide free margin
b enucleation with radiotherapy
c enucleation with bone graft
? implant question about tissue connection to the 14
A Osseoimplantion
*** B Osseointegration
C Osseo implant
?The minimum time for applying topical anesthesia 15
sec 30_A
*************min 1_B
min 2_C
min 3_D
diastolic 80SYSTOLIC and 140 .Mean arterial pressure for pt 16
90a
80b
70c
**100d
?year pt has mild to moderate risk caries what tt 25question about deep pit and fissure in a 17
no tt-A
** Preventive resin restoration -b
restoration 1class -c
d sealant
? question about pt has shallow caries in the enamel only how to tt 18
no ttt-A
**( prr)Preventive resin restoration -b
restoration 1class -c
d sealant
?( like this )question about pt taking bisphosphonate on what affects 19
A Bleeding
************* necroticB bone
? ogetherqueation about instrument that retract both cheek and flap t 20
**Minnesota retactor
he has white non scrapable lesion on lateral side of the ,years ago 3Pt underwent renal transplantation 21
: s the diagnosis'what ,frayed &tongue appears corrugated and has shaggy
Hyperplastic candidiasis -A
leukoplakia idiopathic -B
lichen planus -C
***hairy leukoplakia -D
:Dentist see in CBC 22
HB ,platelet ,WBCs ,RBCs .1
k ,HB ,platelet ,WBCs ,RBCs .2
**Hb ,platelets ,HCT ,HCT ,WBCs ,RBCs .3
calcium ,Hb ,platelets ,HCT ,HCT ,WBCs ,RBCs .4
?+ agland Which is controlled c 23
**Parathyroid
: according to ISO system 1#what is the diameter tip size of GG 24
20.a
30.b
40.c
**50.d
Preventive resin restoration PRR is mainly used to prevent 25
A initial caries lesion
***B caries progression
herapyWhat is the goal of maintenance t 26
**a To prevent current disease
b Evaluate tissue response
? Time for enamel etching in veneer 27
**20-a
30-b
40-c
? question about conscious sedation 28
A unsafe
B uneffective
**C safe and effective
ray-Direction of caries base in x 29
Pulp Towards (A
Towards dentine (B
Towards Enamel (C
***Towards DEJ (D
? Bone resorption around implants after the first year 30
**mm 0.5 .a
mm1 .b
mm 1.5 .C
mm 2.D
s the styrene rubber allow-Modification of complete denture by reinforcing acrylic resin by butadiene 31
.patient to handle the denture more safely
?What do you call this type of dentures
high strength complete denture -a
*** high impact complete denture -b
immediate denture -c
transitional denture -d
?minimum 45for year pt has avulsed anterior mandibular tooth 14 32
Pulpectomy with CaOH then reimplant-A
Reimplant and splint and pulpectomy with CaOH for aweekk .b
****************weeks 3Reimplant and splint and observation for .c
See Translation

Exam31
1. 60 year old patient came to the clinic complaining of excessive movement of denture. On examination
there is elevation of anterior end when u press o the distal end. TTT
A. Reline***
B. Rebase
C. Remake
D. Denture adhesive
2. indirect sequelae of C.D
A. altered taste perception
B. alveolar ridge resorption**
C. oral cancer
3. during jaw relation dr recorded excessive VDO. This will result in following
A. corner lips curved downwards
b. wrinkles around lips
c. excess load on supporting structures**
d. poor retention
4.FOTI of pulp chamber during access preparation ,canal orifice appear
a. light
b. dark***
c. white
d. yellow
5. surfaces visible in FOTI
A. All surfaces of all teeth***
b. all surfaces of anterior teeth
c, proximal surface of anterior
d. proximal surface of posterior
6. histologic and clinical picture of central giant cell granuloma
7. in the panoramic radiograph taken apices of anterior teeth obscured by a shadow. To avoid this what to do
during the reopeat opg
a. chin place up
b. chin down
c. tongue place on palate**
d. lip close around bite block
8. opg of cleidocranial dysplasia
9. radiograph of sickle cell anaemia
10. overlapping of proximal surfaces in xray due
a. incorrect vertical angulation
b. incorrect horizontal angulation**
c.patient move during xray
d. improper placement of film
11. after enamel formation ameloblast forms
A. primary enamel cuticle.**
b. secondary enamel cuticle
c. stratified squamous epithelium
d. degenerates
12. enamel doesnot regenerate once damaged. Why
a. major content organic
b. major content inorganic
c. small amount of organic components
d. formative cells disappear once formed**
13. intraligamentary injection causes pulpal circulation to
a. cease for 30 minutes*************
b. decrease slightly
c. marked decrease
d. no change.
14. patient returns with pain after 1 week of RCT. IOPA reveals RCT failure. Whats your immediate
approach?
a. analgesia**
b. start re rct
c. schedule for apicectomy
d. antibiotic and analgesic
15. a scenario of young patient with repeated case of pericoronitis. But here patient has come with malaise
swelling and severe pain. Your TTT
A. . extract the tooth
b. irrigate with h202 and mouthwash n all
c.operculectomy
d. extract after symptom subsides**
16. something about a patient who is mildly immunocompromised(sorry I dont remember the exact sentence)
antibiotic indicated in which scenario
a. multiple extraction
b. drainage of an abscess**
c.aleolar osteietis
17. story about pregnancy gingival bleeding and red swelling.
a. granuloma gravidarum**
b. purulent something
c. anug
18. for localizing pain doing anesthetic test where to start
a. maxillary right**
b. maxillary left
c.maxillary anterior
d, mandibular right
19.dye that is used with toludene blue to differentiate between cancer and normal cells
a. methylene blue
b. congo red
c.lugol***********
20. patient came for sealing a deep fissure. But dentist not comfortable of sealing a potential caries and
decide to visualize and do the filling.what is this type of restorations with minimum preparations called(not
exactly these words. I felt the question was vague)
a. preventive resin resin restoration **
b. pit n fissure sealant
c. conservative restoration
21. patient with Prosthetic mitral valve and want to do surgical extraction what u do before the appointment
A-prophylactic antibiotic **
B-proceed
22. Cells responsible for Periapical granuloma
a. Neutrophils
b. Plasma
c. Macrophages.**
d. Leucocytes
23. complete mandibular ridge impression that produces excess in the distal end of custom tray :
A-brown compound
B-alginate **
C-polyester
D-additional silicone
24.caries prone patient.dentist prescribe fluoride and ask patient to reduce his carbohydrate intake and use
fluoride mouthwash. Patient returns after 2 days with
a. Gingival enlargement
b. staining
c. dry mouth**
d. no effect
25. After root fracture best tissue healing..
A. Interproximal connective tissue
B.interproximal bone
C.interproximal bone nd connective tissue************
D.inflamed tissue
26.After gingivectomy surface epithelisation occurs in
A.3 days
B.5-14************
C.14-21
D.Over a month
27. True about odontoblastic layer?
a. Filled with odontoblastic processes**************88
b. Characterized by cell-to-cell gap junction
c. Characterized by cell-to-cell tight junction
d. Have higher concentration of cells?
28. Most commonly used intracanal medicament:
a. Ca(OH)**
b. EDTA
c. formacresol
d. naocl
29.Most common feature in diabetics
a. Gum abscess
b. Gingival recession**
c. peridontal pocket
d.mobile teeth
30.What is the most common problem with Ackermans type of clasp
a. Mobility**
b. rigidity
c.detrimental to abutment
31.for optimum apical seal..Where is the spreader used upto
a. beyond apex
b. 1mm short of radiographic apex
c. 1mm short of working length**
d. Till full working length
32. physiologic occlusion
Dont remember the options
33.iopa reveals underobturated rct. U started re rct. But file not going till working length. Not
wetspot(something like that not sure) what d be the reason
a. ledge**
b. perforation
c.fractured instrument
34. irreversible pulpitis is
a. lingering severe pain on cold *************
b. pain on percussion
c.short sharp pain on cold
d. pain on heat
35. indirect pulp capping in
a. pulp has exposed while soft carious dentin
b. pin point bleeding due to fracture
c. caries in dentine*************
36.10 year old patient coming with mobility of upper premolar lower canine.. pockets seen.
a. localized aggressive periodontitis*************
b. generalized aggressive periodontitis
c. papillone lefevre syndrome
d. anug
37.short crown .. to improve resistance and retention
a. many vertical grooves********
b. shallow preparation on the occlusal
c. shoulder finish line
d. make the preparation more parallel
38. unilateral posterior crossbite during erutption of laterals due to
a. functionl shift**
b. improper swallowing habit
c. imbalance between tounge and cheek
d. tongue posture
39. kid with crossbite of lateral incisor. Treatment with Z spring. Best prognosis in
a. . lingually inclined with 50 % vertical overlap
b. linguallly inclined with 5%vertical overlap***************
c. labially inclined with 50% vertical overlap
d. labially inclined with 5% vertical overlap
40- Proxy brush is used with which type of embrasure?
a- type 1
b- type 2**
c- type 3
d- type 4
41. Most suitable to describe NITI
a.Rigidity
b.Low coefficient of friction
c. shape memory**
42. Best method to remove GP from canal for post insertion..
a.Mechanical drill**
b. Chemical solvent
c. Heat
44. Maryland bridge most commonly used in
a. maxillary lateral incisor***********
b. maxillary central incisor
c. mandibular premolar
d. mandibular lateral
45. during metal try in gap between crown and finish line. Best option
a. remake**
b. burnish the metal to margins
c. fill the cap with composite (actually I dont remember exactly)
46. mother worried about partially erupted first molar in child with caries prone mouth.. whats your ttt
a. fluoride application****************
b. pit an fissure sealant
c. ask her to wait till it completely erupts
47.a question about fluoride dosage
48.order of placing of winged clamp type rubber dam
a. clamp before dam
b.clamp aftr dam
c. clamp and dam together*************
49. patient with sensitivity to amide type of LA. What colour catridge would u choose
A. red
b. blue
c. yellow**
d. green
50.The least important to check during complete denture try in:
a- vertical dimension
b- adaptation of margins**************
c- retention
d- centric relation
51.Contraindication for porcelain laminate veneer:
a- fluorosis
b- diastema
c- high lip line
d- bruxism**
52.years old female patient suffering
from pain and discomfort related to the
area covered by this RPD. This problem started 1
month after delivery of this RPD. MW and rinsing
the area with saline improved her condition but the
problem became worse once she wears the
denture again.denture has no occlusal rest.
a- sensitivity to denture material
b- gum stripping design****************
c- no minor connectors
d- bad oral hygiene
53.gates glide gates glidden
a.has no 6-9
b.for coronal preparation**************
c.breaks if stuck
d.for root preparation
54.origin of median border of the lip from
A Fronto nasal process**
B First pharyngeal arch
C Maxillary process
D Mandibular process
55.After ortho treatment xrays shows resorption in middle of root .ttt
A. Caoh2 at the site ofresorption**
B. Extract and reimplant
C. Extract and do implant
D. Do one visit rct
56.special property of GIC
A. Fluoride leaching**
B. Wear resistance
C. Esthetics
57.Which instrument is used subgingivally
A . gracey curette**
b. sickle
c. hoe
58.use of GMT
A. Bevelling of gingival floor of class 2**
B. Removing unsupported enamel rods
C. Class 3 n 5 something
59.Gingivectomy contraindicated in
A. Suprabony pocket
B. Suprabony abcess
C. Osseous reconstruction**
D. Gingival hyperplacia
60.Root canal shape should be
a. Parallel wall closed at apex
b. Converging coronally
c. Converging apically**
61.Osseous crater
a. 1 walled
b. 2 walled**
c. 3 walled
62.Where to hold the suture needle
a. At the end
b. One third from end**
c. Two third from end
d. One third from tip
63.Cutting edge of an ideal instrument should be
a. Parallel to long axis
b. Perpendicuklar to long axis
c. Perpendicular to shank*****************
d. 3 mm from long axis..
64.Caries index
a. DMFt
65.Which of the following induces hyperalgesia in local-nerve fibers?
a. Prostaglandin and serotonin*************
b. Lysosomal enzymes
c. Calcitonin gene-related peptide
d. Substance p
66.Morphology of incisor
A. Cingulum at middle 3rd
B. Labial suface smoothly convex with grooves**
C. Mesi incisal line angle more rounded than distolingual
67.Instrument for carving PPS on cast
A. Kingsley scraper**
B. Lacrons carver
C. Spoon excavator
68.Best initial finish of composite achieved by
a. Carbide bur**
b. Diamond bur
c. White stone
69.One question about secondary trauma from occlusion
70.Autogenous dermal graft are used for tmj disc perforation because
A. Formation of viable dermis and fibrous connective tissue*************
B. Tissue is viable
C. Formation of fibrous connective tissue after degeneration
D. Becomes one with glenoid fossa
71. Drugs commonly causing xerostomia
a. Acetylchoilinesterase inhibitor
b. Opioids
c. Antihistaminics**
72. when to correct thumb sucking
a. as early as possible
b.primary dentition time
c. early mixed dentition************
d, late mixed dentition
73.definition of attachment level
74 Kennedys classification
75.a question about secondary trauma from occlusion
76. instrument number question
77. most important in carbohydrate intake
a. duration
b. frequency*
c. type
d.time
78. most important factor about habit
a. duration **
b.frequency

Exam32
1 .After gingivectomy surface epithelisation occurs in
A.3 days
B.5-14***
C.14-21
D.Over a month
2. 6 years old patient, suers from dental pain, has blue sclera with defective teeth structures,with history of
multiple fractures and shortened gesture:
1.Dentinogenesis imperfecta
2.Amelogenesis imperfecta
3.Osteogenesis imperfecta************
3. Caries susceptible surface in a high caries patient
Facial surfaces of maxillary posteriors*********
Facial surfaces of max anteriors
Palatal surfaces of max posteriors
Palatal surfaces of max anteriors
4 Slowest acting LA
Bupivacaine***
Lignocaine
Mepivacaine
5. During endodontic treatment the file broke ,when we can leave it and have best
prognosis:
a- long part away from working length at early stage of mechanical preparation.
b-short part near the working length at early stage.
c-long part near the working length at late stage *****
d- short part away from working length at late stage.
6.Neonatal teeth;
a- before birth
b- 0-30 month***************
7 Most common salivary gland tumor.. Pleomorphic adenoma
8 Patient with periodontal problem undergoing orthodontic treatment , what type of force will be used?
1- light ********
2- tipping
9 Radiographically in which of the following cysts inverted pear shape appearance is present ?
a) Median palatal cyst
b) Nasoalveolar cyst
c) Globulomaxillary cyst*****
d) Nasopalatine cyst
10.At which age parent shuld go to the orthodontist for consultation:
1.when all permanent teeth erupt
2.when all anterior permanent erupt ******
3.when there is spacing present
4.when there is cowding present
10. Patient has upper complete denture and lower partial missing teeth, with anterior lower teeth.this is
called?
a- combination syndrome**** rest option i forgt..
11.Main role of water irrigation system ?
a) To dilute bacterial toxin***
b) to prevent accumulation bacterial plaque
c) to prevent formation of plaque.
11.During tooth extraction root fracture 1/2 the surgeon do drill inside the root to pull it up with what
instrument
A...crane pick elevator **********drilling is done to createplace to engage th elevator
B..root pick elevator
12. Enamel hypoplasia??
a-apposition ***
b-defection
c-proliferation
13 in deep carious lesion a pin point pulp exposure occur with excessive salivary flow but dentist use a
rubber dam in this condition the treatment will ?
A. pulpotomy
B. pulpectomy
C. direct pulp capping***
D. indirect pulp capping
14 What impression material can result an overextended flange when doing analogue
A) polyether
addition silicone
C) brown compound ******
D) alginate
15 when it's better to use thermoplasticized gutta-percha
gutta :
a. in curved canals
b. in canals with internal regularities.***
c. when lateral condensation is hard
Options were differnt
16.Which not present in saliva ?
Lysosome.
2.potassium k+
3.antibodies.
4.Proteolytic enzymes***
17. RPD that totally supported d by teeth:
class 1
class 2
class 3****
class 4
18 Mostfrequant tooth to use resin bonded fpd
a.max.premolar
b.max.central incisors
c.max lateralincisors***********
d.mand.incisor
19. remodeling of periodontal ligament is by
a osteoblast
b osteoclast
c fibroblast*********
19 reverse line indicate termination of which activity?
a.osteoclast*******
b osteoblast
c fibroblast
d csteocyte
20. What is the difference between radicular cyst and
rarifying osteitis?
a. Vitality
b. Mobility
c. Well-defined borders
d. Being radiolucent vs radio-opaque
opaque ******
options were differnt
21 tooth with large carious there is discoloured
discol dentin,remove it
Large bur with low speed****
Large but with high speed
22. origin and insertion of lateral ptyergoid
23 ,grossly decayed tooth,,,, hw to mange...dnt remember d exact words
post and core , crown lengthning and one mre option,,it was lke sequence what we will do first
24 Difference between Gracey and universal curette:
a. Section of gracey is hemicircular and in universal triangular.
triangular
b. Gracey has one cutting edge while universal has two.
c. Gracey used for cutting in specific area while universal is in any area.
d. Universal 90 not offset, gracey 60 offset.
e. a and d
f. a, b and c.
g. b, c and d. ****
here answer G is correct .
25 primary goal in treatment plan
a.restore all carious lesion
b.eliminate iatrogenic plaque in all patients
c. detect incipient caries and high risk patient
d. complete elimination of plaque from all patients ****
26. The local anesthesia depend on
A)strength bond between drug and nerve***
B) strength bond between drug and its intensity
C) bond between drug and time of removal from body
27./Amalgam for retrograde filling :
Zinc free**
High copper
Mercure free
Antibiotic amalgam
28 Which solution can cause corrosion in the tools?
A- dophor
B- sodium hypochlorite********
29 pt use aspirin 4 times a day what type of test needed
a.bleeding time****
b.coagulation time
30 Main use of dental floss
:a. Remove calculus.
b. Remove over hang.
c. Remove bacterial plaque.***********
d. Remove food debris
31 The smallest part in root canal /
apical constriction**********
a Apical foramen
b Radiographic apex
32 long ques abt invisalign system. option was..
a electric soldering
b spot welding ****
..............
.............. i ckoose b
33 For lower buccal premolar the purpose of inclining the handpicew placed lingually to
a avoid buccal pulp horn****
b avoid lingual pulp horn
c .................
d......................
34. got 10 12 ques on practcal secionero...
35 external fixation.. techniques were der...
36,Skeletal face is from:
1.Neural crest. ***
2.Paraxial mesoderm.
3.Lateral plate mesoderm
37 .Seibert clasification... Apicocoronal resrption in edentuolous patient :
Class ii
38 .taken from one side to another from same patient :autograft
39 Rigid internal fixation is planned in patient who has suffering from isolated zygomatic fracture, the
surgeon wants to use bioresorbable plate to stabilize the fracture, commercially available resorbable plates
are composed of polymer itself reinforcement:
1-Titanium
2- polyglycolic acid ********
40. Long case with a picture: diagnosis was dentigerous cust
41. question on rest.. totaaly difficlt optionns
42. 8 to 9 prostho questions
Exam34
1) Which tooth has to be surgically removed the most :

a) maxillary first molar.


b) mandibular third molar.***
c) impacted canine
d) mesially extruded molars.

2) how much bone resorption is seen around implants after first


year?

a) 1mm
b) 2mm
c) 0.5mm***
d) 1-1.5mm

3) how much flouride is present in commercially available


dentifrices?
a) 500
b)1000***
c)1500
d) 1500

4) A child who lives in an area with water fluoridation 0.4ppm,


how much quantity of flouride in tablet will you give him?

a) 0.25mg***
b) 0.4mg
c) 1mg
d) 2mg

5) Commercially available flouride used for application in dental


surgery?
a) Duraphat ***

(Don't remember other options )

6) Main role of water irrigation system ?

a) To dilute bacterial toxin***


b) to prevent accumulation bacterial plaque
c) to prevent formation of plaque.

7 and 8 )Both Questions described dental pain of long and short


duration in patients and given EPT readings , percussion tests
and sensitivity described towards hot and cold. EPT readings of
these teeth were compared with readings of case control
readings of a vital tooth. Diagnosis asked based on comparative
readings .Options given were:

A) Reversible pulpitis
B) Irreversible pulpitis
C) acute periodontal abscess
D) Normal pulp.

One answer was Reversible pulpitis other was normal pulp


according to EPT readings given.

9) Image of permanent molar in a 9 year old child explained that


fissure intact on exploration with probe,asymptomatic ,white spots
visible, what will be the treatment ?
a) pit and fissure sealant
b) preventive resin restoration
c) restoration.
d) flouride application *************

10) Clinical scenario of 45 year old male who has lesion in oral
cavity. Social History
When taken from him he explained he has been working in the
Sun most of his life?

a) Ameloblastoma
b) keratoacanthoma***
c) squamous cell carcinoma.

10) how much is facial reduction in Veneers?

a) 0.5mm**+
b) 0.7mm
c) 1mm
d) 2mm

11) what are the principles of Guided tissue regeneration ?


a)wound stabilisation and secondary healing.
b) wound stabilisation and primary healing
c) wound stabilisation and making guided surface
biocompatible.***

12) 35 year old man comes for scaling you will use ultatsonic
scaling technique which has epileptic motion ?

a) magnetostrictive***
b) peizoelective
ive
c) can't remember third fourth option

13) A study which is conducted in different cases to find out the


etiology of different diseases varying in different subjects :

A) case control study


B) clinical trial***
C) cross sectional study

ear old woman comes to the clinic with complaint of


14) A 60 year
angular cheilitis and she has been wearing the same denture for
last many years. What is the reason for this?

A) decreased Vertical dimension***


B) over extended denture flange
C)under extended denture flange
f
D) something about wrong eccentric position

15) how can accurate sterilisation be achieved ?

A) by changing strip indicator weekly and spore former indicator


daily
B) by changing strip indicator daily and spore former weekly ***
C) by changing sstrip
trip indicator weekly and spore former monthly

16) Peg laterals develop in which phase of development? (The


word morphology was used in the question I cannot recall the
exact question but since term tooth morphology was used I
Some Qs are answered within the Qs
marked option A correct. Some

themselves.)

A) morphodifferentation***
B) apposition
C) histodifferentation.

17) streptococcus mutans are seen most commonly in :

A) proximal surfaces
B) pits and fissures ***
C) occlusal surfaces

18) patient on warfarin therapy has to go surgery how much


should should blood count be:

A) PT 1-1.5
B) PTT 2-2.5
C) PT 2-2.5***
D) PTT 1-1.5

19) L.A. given to pregnant lady.


A) procaine
B) lidocaine ***

20)edentulous patient with complete dentures came in complains


of commissural cheilitis :

A) decreased VD ***
B) over extended denture flanges.
C) ill fitting denture
D)decreased over jet.

21) patient came to clinic with wrinkled skin and white shiny hair,
with pegged laterals teeth :

A) ectodermal dysplasia***
B) cleidocranial dysplasia
C) pteygz jogheurs syndrome
D) Gardner syndrome

22) clinical scenario of over denture with implants ( image also


shown ) pt had over denture 6 months ago and now he has
complaint of pain under denture area. On examination under
denture area swollen but denture is either under extended or over
extended looks normal what is the cause of pain and swelling

A) over extended denture flange


B) under extended denture flange
C) torsional forces in denture area ***
D) hypertrophy due to ill fitting over denture

23) best way ( least damaging )to remove GP from canal for post
core prep:
A) mechanical( drill)***
B) chemical
C) solvent

24) cleft palate develops in which week of intrauterine life:

A) 6th week
B) 6 to 8th week
C) 10 weeks
D) 9-11 weeks***

25) sequence of restoration :

A) caoh- varnish- ZnPo- Composite


B) varnish - caoh- ZnPo - composite
C) ZnPo - varnish - caoh - composite
D) Caoh- ZnPo - varnish- composite ***

26) rationale of sterilisation :


A) breakage of protein ( protein denaturation)at relatively lower
temperature ***
B) breakage of protein at very high temperature
C)breakage of protein at room temperature

27) MTA superior over other sealer because :


A) more biocompatible
B) high tensile strength
C) high compressive strength***
D)superior sealing properties

28) Forcep used for extraction of maxillary premolars


A) universal no 1

29) during extraction of #14, tooth falls back into oral cavity
which is the best forcep to retrieve tooth :
A) Russian forcep***
B) stillies forcep
C) Addison forcep
D) Ellis forcep

30) clamp used in rubber dam for second primary molar :


A)8A
B) 14A
C) 12WA ****************

31) what do you do before restoring a primary tooth ?


A) Apply varnish
B) dry and isolate cavity only ***
C) apply base first

32) drugs which should not be given together :


A) penicillin with tetracycline

33) why is lab remounting done ?


A) to recheck the correct alignment of teeth ***
B) to check mandibulo maxillary relation of teeth
C) to check vertical dimension
D) to check centric and eccentric relation of teeth
34) which surface of teeth is most vulnerable to caries:
A) mesial surfaces of maxillary molars ***
B) distal surfaces of maxillary molars
C) mesial surfaces of mandibular bicuspid
D) distal surfaces of mandibular bicuspids

35) which teeth are affected most by nursing caries:


A) primary maxillary incisors ***
B) primary mandibular Incisors
C) primary maxillary molars
D)primary mandibular molars

36) Primary teeth affected by which caries at early age :


A) early childhood caries ***
B) nursing caries
C) rampant caries

37) when should be the first visit to the orthodontist :


A) 2-3 years
B) 3-6 years ***
C) 7-8 years
D) 9-10 years

38) pt with mandibular fracture at the neck of mandible


,symptomatic ,surgeon wants to diagnose what kind and extent of
fracture which view is best :

A) antero posterior view


B)reverse town ************
C) sub mento vertex
D) lateral oblique view

39) patient with amelogenesis Imperfecta which bracket will we


use for orthodontic treatment:
A) invasilign

40)patient with resorption / mobility in teeth what kind of force for


orthodontic movement:
A) light ***
B) continuous
C) intermittent
D) heavy

41) best material for implants :


A) titanium ***
B) stainless steel
C) Cr- Co

42) instrument used for levelling line angles , smoothening


gingival seat :
A) enamel hatchet ***
B) chisel
C) hoe
D) bin angled chisel

43) Question about Kennedy class 2 modification 1

44) discoid cleoid instrument used for :


A) carving ***
B) burnishing
C) scrapping off excessive material.

45) composite restoration enamel etching for:


A) 37% phosphoric acid for 10 secs
B) 37% phosphoric acid for 20 secs ***
C) 10 % polyacrylic acid for 30 secs
D) 10% polyacrylic acid for 20 secs

46) Pt root canal treated .he has pain now radiograph reveals
inadequate RCT what is your emergency treatment ?
A) analgesic ***
B) emergency RCT and then antibiotic
C) analgesic and antibiotic

47) which statement best describes progression of caries in teeth


:
A) dentinal tubules are open which lead to bacterial ingress into
Canals ***
B) sorry can't remember rest of the statements

48) how should needles / sharp instrument be disposed off in


surgery ?
A) Specially designed container for sharps.***
B)container for normal waste
C) container not to be burnt.

49) amount of flouride given to patient depends on:


A) age***
B) weight
C) water fluoridation

50) damage to gag reflex which nerve is involved:


A) glossopharyngeal
B) vagus ***
C) trigeminal
D) facial

51) 65 year old patient complains of pain sharp and lingering in


Buccal mucosa. It goes away and comes back spontaneously at
night , stimulated with cold (when going outside ), pt is
edentulous ,has had extractions in the past. Pain is on one side
of face and does not cross the lip line:
A) trigeminal neuralgia ***
B) Bell's palsy
C) myofacial pain.

52) Dentist injects patient with L.A. and suddenly you see bluish
colour swelling on one side of the face what is it :
A) hematoma***
B) fibroma
C) vasovagal attack
D) L.A. Toxicity

53) function of minor connector


A) to resist lateral forces to the denture base
B) to transfer functional stress to abutment ***
C) connects rest seat with retainer at an acute angle.
D) connects rest seat with retainer at 90 degree angle.

54) Bone death occurs in what time :


A) 42 c for 10 secs
B) 42c for 30 secs
3)47c for 30 secs
4) 47c for 1 minute.***

55) Needle used for FNAC : ( fine needle aspiration cytology)


19 gauge.

56) which patient will best tolerate new denture:


A) philosophic***
B) indifferent
C) hysterical
D) Exacting

57)Question about Cemental dysplasia. Clinical scenario


presentation.

58) evaluation of improved periodontal health:


A) no bleeding on probing
B) attachment level***
C) decreased plaque score.

59) 4 year old child has trauma to anterior teeth ,reports in half an
hour to clinic, pinpoint exposure ,sensitivity ,no radiographic
changes, light pain ,slight bleeding, what is the best treatment to
do ?

A) pulpectomy
B) pulpotomy ***
C) apexification
D) apexogenesis
60) Which root seldom has two canals :
A) MB root of upper molar
B) MB root of lower molar
C) Maxillary central incisor
D) Distal root of upper molar ***

61) Patient with osteoradionecrosis how can extraction be done


?

Under hyperbaric oxygen.

62) apical foramen develops from :

A) Epithelial rest of malassez


B) Hertwig root sheath***
C) Rests of Serres.

Exam35

1.zinc polycarboxylate cement better than zinc phosphate-


biocompatibility*****
2.instrument used to plane facial and lingual walls in class 2
Enamel hatchet**********
3.most destructive occlusal interference
Non working side*************
4.pulpal pressure
10mmhg************
5.pt 9 yr old had concussion injury to maxillary central incisor 15
mnths before .no clinical or radiographic findings and no
pain.treatment?
Extraction
Nothing wait and watch******************
Apexification
6.which periodontal fiber is affected first during extraction of
upper first molar
Circular
Transseptal
Dentogingival***********************
7. What us the goal of maintenance therapy
To prevent reccurance of the disease***************
8.child with abscess in the canine region no crowding ............
Extract both canine without space maintainer**********
9.dyring crown preparation if the occlusal reduction is nt proper
Fracture of functional cusp
Occlusal interference*****************
10.prophylactic antibiotic needed
Banding ***********
Bonding
Debonding
11.low caries prone mouth in a child .on examination the Explorer
wedges to pit and fissure in newly errupted molar .treatment
Pit and fissure******************
Flouride
Composite restoration
Amalgam restoration
12.pic of fibroma
14. Pic of osteoma
15.dental floss used
Used to remove interproximal plaque*********
16.brushing for maintaining good oral hygiene************
17.blue hue with obliterated pulp
dentinogenesis imperfect**************
18.palmar and plantar keratosis
papillon levefre syndrome***************
19 . advantage of retention form
Increase strength of restoration
Prevent upward displacement of restoration**************
Prevent tissueward displacement
20.4th pharyngeal arch gives rise to
Superior parathyroid***********
21.intracoronal bleaching is
sodium perborate*********
22.most effective irrigants
Saline
Sodium hypochlorite *******************
Hydrogen peroxide
23.how to measure the working length
Bisecting radiograph**************
Calculate the length of root
Curve the tip of instrument and place into canal
24.environmental factors which cause caries
Flouride ,nutrition*******************
Gingival recession
Dry mouth
Lactobacillus,streptococcus, plaque
25.after rct which post cause less fracture
Fibre post causes less fracture of tooth
26.in smoker presence of gingivitis
Less intense than non smokers*************
27.to sterilise bur
2% sodium nitrite****************
28.caries prone surface
Buccal surface of maxillary posteriors*****************
29.sharpening angle with stone
100-110
30.bacteria in periodontitis
Gram negative anaerobes A. A and P gingivalis
31. Bacteria not present in pericoronitis
staphlococuus
32.solitary cyst treatment curettage and close
33.maryland bridge -resin bonded
34.initiation and proliferation
Amelogenesis imperfecta
Dentinogenesis imperfecta
Oligodontia****************
Hypoplasia

35-bacteria in deep root canal are obligate anaerobe


36-ISO instrument depends upon width of tip of the file
37-early loss of deciduos teeth is seen in pappilon leferve
disease

Exam 36
1-nerve supply of levattor superior palpebrae
Occulomoter
2-intraligamentray injection is for a specific tooth
3-phophate increases the absorption of fluoride
4-morphology of TMJ best seen in CBCT
5-caries progression is at slower rate in adults as compared to
young patient
6-material used for preventive resin restoration is flowable
composite
7-manifestation of pericoronitis acute inflammation around
erupted 3rd molar
8-mechanism of growth at condyle is
Apposition and endochondral
8-access opening for lower 6
Trapezoidal
9-the result pf smear layer removal
Bettrr adaptaion of filling material
10-smear layer benefit
Prevent the toxins to reach pulp through dentinal tubules
11- blood supply of the lower face
Facial artery
12-apical root resorption is due to magnitude of the force
13-gic not used with mouth breathers
14-extra oral block when touchh pterygoid moved upward and
posterior
15-max lidocain dose for a child is 4.4 mg per kg
16-substantivity property is seen in chlorhexidine
17-the longest recommended time for avulsed tooth is 1 hr
1/what increase the absorption of fluoride in the body ??
a. ca
b. mg
c. phosphate*****
d. ?
2/Anesthetic test (intraligamentary) used to localize the pain:
A. in specific tooth***
B. from upper and lower
C. from right to left in maxilla
D. in anterior teeth
3/Which of the following is the most approprite for determining the
morphology of the tmj
A. MRI
b. Cbct ********************
C ct
4/.Careis in old patient compared to younge patient :
a.Progress slowly in adult ******************
b.Adult suffers less pain compaired to young
c.More progreesing in old patient than young
Same like that but options not same but near to it
5/which material is used during preventive composite restoration
as long fissure sealing :
Compomer
Flowable composite*****************
Glassinomer
Conventional composite
6/in standard files how to classify?
A.length of file
B-size of hand instrument
C.width of tip instrument*******************
7/Manifestation of Pericoronitis:
A) Swollen Suppurative
B) Acute inflammation around erupted 3rd molar ******************
Com 4 me n other option I choose b
8/Mechanism of mandible growth at condyle
Apposition and intramembranous modeling
Interstitial and endochondral
Apposition and endochondral **************
Interstitial and intra membranous
9/Nerve supply levator suoerio paloebral
occulomoter
10/Ques about after pulpectomy wat restorative material in canal
zinc oxide euginol
11/The result of smear layer removal?
Increase bacterial toxins passage
Dec permeability of dentine
Better adaptation of filling material*****************
12/2 question about titanium
13/ smear layer benefit in protecting pulp through:
a- prevent toxins to reach pulp through dentinal tubules
b. decrease acid etch effect on pulp
Also wz another option I choose a
14/the common goal for periodontal flap is :
A. Remove granulation tissue
B. Access to diseased root surface*************
C. Adjust mucogingival prep.
D. All of the above
Com to me but not same option but answer shud be b
15/After crown lengthening you wait for 6 months before placing
crown for
Maturation of periodontium**************
New Junctional epithelium formation
16/Since cavity deisn for composite shud be conservative as
much as possible ; concept is acceptable
Conventional amalgam
Bevel amalgam
Extension for prevention
Conservative prep according lesion extension**************
17 /about scalp layer on which haemorrhage go along cranium
Subcutaneous ct
Subperiosreal****************
Some thing like that I can't remember exactly but u shud to know
layer of scalp
18/Wax grooving in wax patterns
Pk2
Pk4 *************
Spoone excavator
19/restoration not used wz mouth breathing pt??? Gi
20/Apical Root resorption due to force of magnitude
21/Quest of extraoral nerve block of mandibular nerve
Direction of needle after u touch ptreygoid plate?
Upward and posterior
22/Young patient with large caries on the lower 6 unrestorable
when thedr should extract this tooth :
a-after complete eruption of the 2nd premolar
b-after comlpete eruption on 7
c-after 7 reach bifurcation area****
d-after complete eruption of 8
23-Symptomatic periodontitis compared to asymtomatic :
a.Former causes pain because it is dominated by enterococcal
feacalis .****
b.Later is dominated by streptcoccus .. Cant remember exactly
the name of bacteria mentioned in this option ,, sorry .
c.Fungal infection in both
24/Clean instrument before entering the sterilization why
a. It helps to kill bacteria
b. speed sterilization process
c. for integrity of instrument
D. Removal of porion protein) ****
25/Smokers : Oral cancer
Control
Smokers
60%
40%
Non smokers
10%
90%
What is the percentage?
Adds ratio 90x60/10x40 = 5400/400 = 13.5***
26/Malignant Timor in upper jaw n b4 surgery prosthodontic
consideration shud be taken wat the best to be done in surgery to
enhance obturator function
Pic was upper jaw wz large defect in ridge after removal of lesion
1 sacrifices as much as possible of soft palate
2 removal of bone from palate shorter than ridge bone removal
3 use dermal graft in wall of defect ****************
4 removal of teeth mesial to defect
27/Antibiotic that cause cell wall destruction:
A penicillin************
B.Tetracycline
C.sulfonamide
D.Clindamycin
28/Local anasthesia administerd according to
Nerve diameter
Extension of operative site*****************
29/Established gingivitis TAKE HOW MANY DAYS
14-21 days
30/Pic of nasopalatine cyst
31/Also radicular cyst
32/Max dose of lidocaine to child
34/The best description of(healed) asymptomatic periodontitis
Disappear of sinus
Decrease lesion
Disappear of lesion
Tooth bcome asymptomatic *****************
-
35/Quest the most biocombatible cement for pdl n pulp???
gic
36/Another quest about DMFT
Disadvantage of it overestimation of caries n affect
internal ,Validity ******************
external,Validity
Internal reability
External reability
71/-Case of wz lesion in inner surface of lower lip n history of
trauma ?? Fibroma
72/Water fluoridation 1ppm
73-Case about irreversible pulpitis
74/Cherubism quest
75/Main blood supply to floor of mouth
Lingual artery
76/pt fracture upper denture in midline from ant to pps n say it
was broken b4 2 or 3 times n by examination presence of
inflammation on residual ridge n sever bone resorption Wat
cause of fracture
I'll fitting denture n thick frenum
Unbalanced occlusion *****************
77/pt wz skeletal class 3 need upper n lower complete denture
management
Set upper teeth in large mold
Set lower teeth in small mold
Set teeth in cross bite ************************8
,removal of premolar from the large jaw
78/flouride used as varnish
duraphat
83/clamp used wz partial erupted molar permenant
14A

85/case n say ossifying fibroma around lower molar tooth Wat ur


management n itis discover during check up n no bone expansion
n vital tooth
curretage
86/-forcep of epulis fissuratume
Allis forcep
87/bone interproximal parallel to gingival pdl cej ,,,
88/shwartiez to remove tip of currate
89/case 51 woman wz denture complains from buring sensation
wz xerostomia n denture ok wzout problem taste sensation
change n disturbance in sleep wz test blood of ca n hp n vit d I
read all below normal range but not more
Diagnosis neuralgia
bells palsy
menupose ***********************
viral infection
90/case of pagets
92/stainless steel crown n ready to cementation but u find very
small open in margin from buccal Wat d u do
Crimbing wz plier 112
Crimbing wz plier 114**************
Another third no of plier n forget
Use another one crown

93-guiding planes are placed on the


Distal*****************
Mesial
94-rarefurcation is
Areas denuded of bone
95-Impression material for [patient taking radiotherapy
Alginate
96-most common disc displacement is
Anteriorly
97-size of rest seat vertically
1.5
98-C shaped canal seen in
mandibular second molar
99-occlusion of primary teeth depends upon
Terminal plane of molars
100-growth center between bones of skull
Sphenooccipital synchondrosis

Вам также может понравиться